SlideShare a Scribd company logo
1 of 95
Download to read offline
Section 5.5
           Integration by Substitution

                    V63.0121.041, Calculus I

                        New York University


                       December 13, 2010



Announcements
   ”Wednesday”, December 15: Review, Movie
   Monday, December 20, 12:00pm–1:50pm: Final Exam
Announcements




        ”Wednesday”, December 15:
        Review, Movie
        Monday, December 20,
        12:00pm–1:50pm: Final
        Exam




V63.0121.041, Calculus I (NYU)   Section 5.5 Integration by Substitution   December 13, 2010   2 / 37
Resurrection Policy
 If your final score beats your midterm score, we will add 10% to its weight,
 and subtract 10% from the midterm weight.




Image credit: Scott Beale / Laughing Squid
   V63.0121.041, Calculus I (NYU)          Section 5.5 Integration by Substitution   December 13, 2010   3 / 37
Objectives



         Given an integral and a
         substitution, transform the
         integral into an equivalent
         one using a substitution
         Evaluate indefinite integrals
         using the method of
         substitution.
         Evaluate definite integrals
         using the method of
         substitution.




 V63.0121.041, Calculus I (NYU)   Section 5.5 Integration by Substitution   December 13, 2010   4 / 37
Outline



Last Time: The Fundamental Theorem(s) of Calculus


Substitution for Indefinite Integrals
   Theory
   Examples


Substitution for Definite Integrals
   Theory
   Examples




 V63.0121.041, Calculus I (NYU)   Section 5.5 Integration by Substitution   December 13, 2010   5 / 37
Differentiation and Integration as reverse processes


Theorem (The Fundamental Theorem of Calculus)

 1. Let f be continuous on [a, b]. Then
                                                    x
                                        d
                                                        f (t) dt = f (x)
                                        dx      a

 2. Let f be continuous on [a, b] and f = F for some other function F .
    Then
                                        b
                                            f (x) dx = F (b) − F (a).
                                    a




 V63.0121.041, Calculus I (NYU)   Section 5.5 Integration by Substitution   December 13, 2010   6 / 37
Techniques of antidifferentiation?



So far we know only a few rules for antidifferentiation. Some are general,
like
                [f (x) + g (x)] dx = f (x) dx + g (x) dx




 V63.0121.041, Calculus I (NYU)   Section 5.5 Integration by Substitution   December 13, 2010   7 / 37
Techniques of antidifferentiation?



So far we know only a few rules for antidifferentiation. Some are general,
like
                [f (x) + g (x)] dx = f (x) dx + g (x) dx

Some are pretty particular, like
                                     1
                                   √       dx = arcsec x + C .
                                  x x2 − 1




 V63.0121.041, Calculus I (NYU)    Section 5.5 Integration by Substitution   December 13, 2010   7 / 37
Techniques of antidifferentiation?



So far we know only a few rules for antidifferentiation. Some are general,
like
                [f (x) + g (x)] dx = f (x) dx + g (x) dx

Some are pretty particular, like
                                     1
                                   √       dx = arcsec x + C .
                                  x x2 − 1
What are we supposed to do with that?




 V63.0121.041, Calculus I (NYU)    Section 5.5 Integration by Substitution   December 13, 2010   7 / 37
No straightforward system of antidifferentiation



So far we don’t have any way to find
                                                   2x
                                              √          dx
                                                  x2 + 1
or
                                                tan x dx.




 V63.0121.041, Calculus I (NYU)   Section 5.5 Integration by Substitution   December 13, 2010   8 / 37
No straightforward system of antidifferentiation



So far we don’t have any way to find
                                                   2x
                                              √          dx
                                                  x2 + 1
or
                                                tan x dx.

Luckily, we can be smart and use the “anti” version of one of the most
important rules of differentiation: the chain rule.




 V63.0121.041, Calculus I (NYU)   Section 5.5 Integration by Substitution   December 13, 2010   8 / 37
Outline



Last Time: The Fundamental Theorem(s) of Calculus


Substitution for Indefinite Integrals
   Theory
   Examples


Substitution for Definite Integrals
   Theory
   Examples




 V63.0121.041, Calculus I (NYU)   Section 5.5 Integration by Substitution   December 13, 2010   9 / 37
Substitution for Indefinite Integrals



Example
Find
                                                   x
                                             √               dx.
                                                 x2    +1




 V63.0121.041, Calculus I (NYU)   Section 5.5 Integration by Substitution   December 13, 2010   10 / 37
Substitution for Indefinite Integrals



Example
Find
                                                   x
                                             √               dx.
                                                 x2    +1

Solution
Stare at this long enough and you notice the the integrand is the
derivative of the expression 1 + x 2 .




 V63.0121.041, Calculus I (NYU)   Section 5.5 Integration by Substitution   December 13, 2010   10 / 37
Say what?


Solution (More slowly, now)
Let g (x) = x 2 + 1.




 V63.0121.041, Calculus I (NYU)   Section 5.5 Integration by Substitution   December 13, 2010   11 / 37
Say what?


Solution (More slowly, now)
Let g (x) = x 2 + 1. Then g (x) = 2x and so

                          d                       1                            x
                                  g (x) =                  g (x) = √
                          dx                 2     g (x)                      x2   +1




 V63.0121.041, Calculus I (NYU)     Section 5.5 Integration by Substitution             December 13, 2010   11 / 37
Say what?


Solution (More slowly, now)
Let g (x) = x 2 + 1. Then g (x) = 2x and so

                          d                         1                                x
                                   g (x) =                   g (x) = √
                          dx                   2     g (x)                       x2      +1

Thus
                               x                        d
                         √            dx =                      g (x)           dx
                             x2 + 1                     dx
                                           =       g (x) + C =                  1 + x2 + C .




 V63.0121.041, Calculus I (NYU)       Section 5.5 Integration by Substitution                 December 13, 2010   11 / 37
Leibnizian notation FTW


Solution (Same technique, new notation)
Let u = x 2 + 1.




 V63.0121.041, Calculus I (NYU)   Section 5.5 Integration by Substitution   December 13, 2010   12 / 37
Leibnizian notation FTW


Solution (Same technique, new notation)
                                                                            √
Let u = x 2 + 1. Then du = 2x dx and                        1 + x2 =         u.




 V63.0121.041, Calculus I (NYU)   Section 5.5 Integration by Substitution         December 13, 2010   12 / 37
Leibnizian notation FTW


Solution (Same technique, new notation)
                                                                               √
Let u = x 2 + 1. Then du = 2x dx and                           1 + x2 =         u. So the integrand
becomes completely transformed into
                                                     1
                              √
                                  x dx               2 du
                                                     √
                                                                       1
                                                                       √ du
                                         =                   =
                                  x2 + 1                u             2 u




 V63.0121.041, Calculus I (NYU)      Section 5.5 Integration by Substitution        December 13, 2010   12 / 37
Leibnizian notation FTW


Solution (Same technique, new notation)
                                                                               √
Let u = x 2 + 1. Then du = 2x dx and                           1 + x2 =         u. So the integrand
becomes completely transformed into
                                                     1
                              √
                                  x dx               2 du
                                                     √
                                                                       1
                                                                       √ du
                                         =                   =
                                  x2 + 1                u             2 u
                                                    1 −1/2
                                           =        2u     du




 V63.0121.041, Calculus I (NYU)      Section 5.5 Integration by Substitution        December 13, 2010   12 / 37
Leibnizian notation FTW


Solution (Same technique, new notation)
                                                                               √
Let u = x 2 + 1. Then du = 2x dx and                           1 + x2 =         u. So the integrand
becomes completely transformed into
                                                     1
                              √
                                  x dx               2 du
                                                     √
                                                                       1
                                                                       √ du
                                         =                   =
                                  x2 + 1                u             2 u
                                                    1 −1/2
                                           =        2u     du
                                               √
                                           =       u+C =              1 + x2 + C .




 V63.0121.041, Calculus I (NYU)      Section 5.5 Integration by Substitution        December 13, 2010   12 / 37
Useful but unsavory variation

Solution (Same technique, new notation, more idiot-proof)
                                              √
Let u = x 2 + 1. Then du = 2x dx and 1 + x 2 = u. “Solve for dx:”
                                                        du
                                              dx =
                                                        2x




 V63.0121.041, Calculus I (NYU)   Section 5.5 Integration by Substitution   December 13, 2010   13 / 37
Useful but unsavory variation

Solution (Same technique, new notation, more idiot-proof)
                                              √
Let u = x 2 + 1. Then du = 2x dx and 1 + x 2 = u. “Solve for dx:”
                                                              du
                                                    dx =
                                                              2x
So the integrand becomes completely transformed into
                                   x                       x du
                            √               dx =          √ ·
                                  x2   +1                   u 2x




 V63.0121.041, Calculus I (NYU)         Section 5.5 Integration by Substitution   December 13, 2010   13 / 37
Useful but unsavory variation

Solution (Same technique, new notation, more idiot-proof)
                                              √
Let u = x 2 + 1. Then du = 2x dx and 1 + x 2 = u. “Solve for dx:”
                                                              du
                                                    dx =
                                                              2x
So the integrand becomes completely transformed into
                                   x                       x du                    1
                            √               dx =          √ ·    =                 √ du
                                  x2   +1                   u 2x                  2 u




 V63.0121.041, Calculus I (NYU)         Section 5.5 Integration by Substitution      December 13, 2010   13 / 37
Useful but unsavory variation

Solution (Same technique, new notation, more idiot-proof)
                                              √
Let u = x 2 + 1. Then du = 2x dx and 1 + x 2 = u. “Solve for dx:”
                                                              du
                                                    dx =
                                                              2x
So the integrand becomes completely transformed into
                                   x                       x du                    1
                            √               dx =          √ ·    =                 √ du
                                  x2   +1                   u 2x                  2 u
                                                          1 −1/2
                                                 =        2u     du




 V63.0121.041, Calculus I (NYU)         Section 5.5 Integration by Substitution      December 13, 2010   13 / 37
Useful but unsavory variation

Solution (Same technique, new notation, more idiot-proof)
                                              √
Let u = x 2 + 1. Then du = 2x dx and 1 + x 2 = u. “Solve for dx:”
                                                              du
                                                    dx =
                                                              2x
So the integrand becomes completely transformed into
                                   x                       x du                    1
                            √               dx =          √ ·    =                 √ du
                                  x2   +1                   u 2x                  2 u
                                                          1 −1/2
                                                 =        2u     du
                                                     √
                                                 =       u+C =              1 + x2 + C .




 V63.0121.041, Calculus I (NYU)         Section 5.5 Integration by Substitution      December 13, 2010   13 / 37
Useful but unsavory variation

Solution (Same technique, new notation, more idiot-proof)
                                              √
Let u = x 2 + 1. Then du = 2x dx and 1 + x 2 = u. “Solve for dx:”
                                                              du
                                                    dx =
                                                              2x
So the integrand becomes completely transformed into
                                   x                       x du                    1
                            √               dx =          √ ·    =                 √ du
                                  x2   +1                   u 2x                  2 u
                                                          1 −1/2
                                                 =        2u     du
                                                     √
                                                 =       u+C =              1 + x2 + C .


Mathematicians have serious issues with mixing the x and u like this.
However, I can’t deny that it works.
 V63.0121.041, Calculus I (NYU)         Section 5.5 Integration by Substitution      December 13, 2010   13 / 37
Theorem of the Day

Theorem (The Substitution Rule)
If u = g (x) is a differentiable function whose range is an interval I and f
is continuous on I , then

                                  f (g (x))g (x) dx =                f (u) du

That is, if F is an antiderivative for f , then

                                   f (g (x))g (x) dx = F (g (x))

In Leibniz notation:
                                            du
                                    f (u)      dx =             f (u) du
                                            dx


 V63.0121.041, Calculus I (NYU)    Section 5.5 Integration by Substitution      December 13, 2010   14 / 37
A polynomial example


Example
Use the substitution u = x 2 + 3 to find                       (x 2 + 3)3 4x dx.




 V63.0121.041, Calculus I (NYU)   Section 5.5 Integration by Substitution    December 13, 2010   15 / 37
A polynomial example


Example
Use the substitution u = x 2 + 3 to find                        (x 2 + 3)3 4x dx.


Solution
If u = x 2 + 3, then du = 2x dx, and 4x dx = 2 du. So

                           (x 2 + 3)3 4x dx




 V63.0121.041, Calculus I (NYU)    Section 5.5 Integration by Substitution    December 13, 2010   15 / 37
A polynomial example


Example
Use the substitution u = x 2 + 3 to find                       (x 2 + 3)3 4x dx.


Solution
If u = x 2 + 3, then du = 2x dx, and 4x dx = 2 du. So

                           (x 2 + 3)3 4x dx =           u 3 2du = 2         u 3 du




 V63.0121.041, Calculus I (NYU)   Section 5.5 Integration by Substitution       December 13, 2010   15 / 37
A polynomial example


Example
Use the substitution u = x 2 + 3 to find                       (x 2 + 3)3 4x dx.


Solution
If u = x 2 + 3, then du = 2x dx, and 4x dx = 2 du. So

                           (x 2 + 3)3 4x dx =           u 3 2du = 2         u 3 du
                                                1
                                               = u4
                                                2




 V63.0121.041, Calculus I (NYU)   Section 5.5 Integration by Substitution       December 13, 2010   15 / 37
A polynomial example


Example
Use the substitution u = x 2 + 3 to find                       (x 2 + 3)3 4x dx.


Solution
If u = x 2 + 3, then du = 2x dx, and 4x dx = 2 du. So

                           (x 2 + 3)3 4x dx =           u 3 2du = 2         u 3 du
                                                1     1
                                               = u 4 = (x 2 + 3)4
                                                2     2




 V63.0121.041, Calculus I (NYU)   Section 5.5 Integration by Substitution       December 13, 2010   15 / 37
A polynomial example, by brute force


Compare this to multiplying it out:

                 (x 2 + 3)3 4x dx




 V63.0121.041, Calculus I (NYU)   Section 5.5 Integration by Substitution   December 13, 2010   16 / 37
A polynomial example, by brute force


Compare this to multiplying it out:

                 (x 2 + 3)3 4x dx =            x 6 + 9x 4 + 27x 2 + 27 4x dx




 V63.0121.041, Calculus I (NYU)   Section 5.5 Integration by Substitution   December 13, 2010   16 / 37
A polynomial example, by brute force


Compare this to multiplying it out:

                 (x 2 + 3)3 4x dx =            x 6 + 9x 4 + 27x 2 + 27 4x dx

                                    =          4x 7 + 36x 5 + 108x 3 + 108x dx




 V63.0121.041, Calculus I (NYU)   Section 5.5 Integration by Substitution   December 13, 2010   16 / 37
A polynomial example, by brute force


Compare this to multiplying it out:

                 (x 2 + 3)3 4x dx =            x 6 + 9x 4 + 27x 2 + 27 4x dx

                                    =          4x 7 + 36x 5 + 108x 3 + 108x dx
                                     1
                                    = x 8 + 6x 6 + 27x 4 + 54x 2
                                     2




 V63.0121.041, Calculus I (NYU)   Section 5.5 Integration by Substitution   December 13, 2010   16 / 37
A polynomial example, by brute force


Compare this to multiplying it out:

                 (x 2 + 3)3 4x dx =            x 6 + 9x 4 + 27x 2 + 27 4x dx

                                    =          4x 7 + 36x 5 + 108x 3 + 108x dx
                                     1
                                    = x 8 + 6x 6 + 27x 4 + 54x 2
                                     2
Which would you rather do?




 V63.0121.041, Calculus I (NYU)   Section 5.5 Integration by Substitution   December 13, 2010   16 / 37
A polynomial example, by brute force


Compare this to multiplying it out:

                 (x 2 + 3)3 4x dx =            x 6 + 9x 4 + 27x 2 + 27 4x dx

                                    =          4x 7 + 36x 5 + 108x 3 + 108x dx
                                     1
                                    = x 8 + 6x 6 + 27x 4 + 54x 2
                                     2
Which would you rather do?
      It’s a wash for low powers




 V63.0121.041, Calculus I (NYU)   Section 5.5 Integration by Substitution   December 13, 2010   16 / 37
A polynomial example, by brute force


Compare this to multiplying it out:

                 (x 2 + 3)3 4x dx =            x 6 + 9x 4 + 27x 2 + 27 4x dx

                                    =          4x 7 + 36x 5 + 108x 3 + 108x dx
                                     1
                                    = x 8 + 6x 6 + 27x 4 + 54x 2
                                     2
Which would you rather do?
      It’s a wash for low powers
      But for higher powers, it’s much easier to do substitution.




 V63.0121.041, Calculus I (NYU)   Section 5.5 Integration by Substitution   December 13, 2010   16 / 37
Compare

We have the substitution method, which, when multiplied out, gives
                               1
            (x 2 + 3)3 4x dx = (x 2 + 3)4
                               2
                               1 8
                             =   x + 12x 6 + 54x 4 + 108x 2 + 81
                               2
                               1                            81
                             = x 8 + 6x 6 + 27x 4 + 54x 2 +
                               2                             2

and the brute force method

                              1
            (x 2 + 3)3 4x dx = x 8 + 6x 6 + 27x 4 + 54x 2
                              2
Is there a difference? Is this a problem?


 V63.0121.041, Calculus I (NYU)   Section 5.5 Integration by Substitution   December 13, 2010   17 / 37
Compare

We have the substitution method, which, when multiplied out, gives
                               1
            (x 2 + 3)3 4x dx = (x 2 + 3)4 + C
                               2
                               1 8
                             =   x + 12x 6 + 54x 4 + 108x 2 + 81 + C
                               2
                               1                            81
                             = x 8 + 6x 6 + 27x 4 + 54x 2 +    +C
                               2                             2

and the brute force method

                              1
            (x 2 + 3)3 4x dx = x 8 + 6x 6 + 27x 4 + 54x 2 + C
                              2
Is there a difference? Is this a problem? No, that’s what +C means!


 V63.0121.041, Calculus I (NYU)   Section 5.5 Integration by Substitution   December 13, 2010   17 / 37
A slick example


Example
Find       tan x dx.




 V63.0121.041, Calculus I (NYU)   Section 5.5 Integration by Substitution   December 13, 2010   18 / 37
A slick example


Example
                                              sin x
Find       tan x dx. (Hint: tan x =                 )
                                              cos x




 V63.0121.041, Calculus I (NYU)   Section 5.5 Integration by Substitution   December 13, 2010   18 / 37
A slick example


Example
                                              sin x
Find       tan x dx. (Hint: tan x =                 )
                                              cos x

Solution
Let u = cos x . Then du = − sin x dx . So

                                          sin x
                         tan x dx =             dx
                                          cos x




 V63.0121.041, Calculus I (NYU)   Section 5.5 Integration by Substitution   December 13, 2010   18 / 37
A slick example


Example
                                              sin x
Find       tan x dx. (Hint: tan x =                 )
                                              cos x

Solution
Let u = cos x . Then du = − sin x dx . So

                                          sin x
                         tan x dx =             dx
                                          cos x




 V63.0121.041, Calculus I (NYU)   Section 5.5 Integration by Substitution   December 13, 2010   18 / 37
A slick example


Example
                                              sin x
Find       tan x dx. (Hint: tan x =                 )
                                              cos x

Solution
Let u = cos x . Then du = − sin x dx . So

                                          sin x
                         tan x dx =             dx
                                          cos x




 V63.0121.041, Calculus I (NYU)   Section 5.5 Integration by Substitution   December 13, 2010   18 / 37
A slick example


Example
                                              sin x
Find       tan x dx. (Hint: tan x =                 )
                                              cos x

Solution
Let u = cos x . Then du = − sin x dx . So

                                          sin x                        1
                         tan x dx =             dx = −                   du
                                          cos x                        u




 V63.0121.041, Calculus I (NYU)   Section 5.5 Integration by Substitution     December 13, 2010   18 / 37
A slick example


Example
                                              sin x
Find       tan x dx. (Hint: tan x =                 )
                                              cos x

Solution
Let u = cos x . Then du = − sin x dx . So

                                       sin x                           1
                         tan x dx =           dx = −                     du
                                       cos x                           u
                                  = − ln |u| + C




 V63.0121.041, Calculus I (NYU)   Section 5.5 Integration by Substitution     December 13, 2010   18 / 37
A slick example


Example
                                              sin x
Find       tan x dx. (Hint: tan x =                 )
                                              cos x

Solution
Let u = cos x . Then du = − sin x dx . So

                                       sin x                           1
                         tan x dx =           dx = −                     du
                                       cos x                           u
                                  = − ln |u| + C
                                  = − ln | cos x| + C = ln | sec x| + C




 V63.0121.041, Calculus I (NYU)   Section 5.5 Integration by Substitution     December 13, 2010   18 / 37
Can you do it another way?

Example
                                              sin x
Find       tan x dx. (Hint: tan x =                 )
                                              cos x




 V63.0121.041, Calculus I (NYU)   Section 5.5 Integration by Substitution   December 13, 2010   19 / 37
Can you do it another way?

Example
                                              sin x
Find       tan x dx. (Hint: tan x =                 )
                                              cos x

Solution
                                                                         du
Let u = sin x. Then du = cos x dx and so dx =                                 .
                                                                        cos x




 V63.0121.041, Calculus I (NYU)   Section 5.5 Integration by Substitution         December 13, 2010   19 / 37
Can you do it another way?

Example
                                                  sin x
Find       tan x dx. (Hint: tan x =                     )
                                                  cos x

Solution
                                                                             du
Let u = sin x. Then du = cos x dx and so dx =                                     .
                                                                            cos x
                                         sin x          u     du
                    tan x dx =                 dx =
                                         cos x        cos x cos x
                                          u du         u du                            u du
                                  =             =              =
                                         cos 2x     1 − sin2 x                        1 − u2
At this point, although it’s possible to proceed, we should probably back
up and see if the other way works quicker (it does).

 V63.0121.041, Calculus I (NYU)       Section 5.5 Integration by Substitution         December 13, 2010   19 / 37
For those who really must know all

Solution (Continued, with algebra help)
Let y = 1 − u 2 , so dy = −2u du. Then
                                 u du         u dy
            tan x dx =                2
                                         =
                                1−u           y −2u
                               1    dy       1                 1
                          =−             = − ln |y | + C = − ln 1 − u 2 + C
                               2     y       2                 2
                                   1                      1
                          = ln √          + C = ln                +C
                                 1 − u2                1 − sin2 x
                                  1
                          = ln          + C = ln |sec x| + C
                               |cos x|


There are other ways to do it, too.

 V63.0121.041, Calculus I (NYU)   Section 5.5 Integration by Substitution   December 13, 2010   20 / 37
Outline



Last Time: The Fundamental Theorem(s) of Calculus


Substitution for Indefinite Integrals
   Theory
   Examples


Substitution for Definite Integrals
   Theory
   Examples




 V63.0121.041, Calculus I (NYU)   Section 5.5 Integration by Substitution   December 13, 2010   21 / 37
Substitution for Definite Integrals


Theorem (The Substitution Rule for Definite Integrals)
If g is continuous and f is continuous on the range of u = g (x), then
                                  b                                    g (b)
                                      f (g (x))g (x) dx =                      f (u) du.
                              a                                      g (a)




 V63.0121.041, Calculus I (NYU)          Section 5.5 Integration by Substitution           December 13, 2010   22 / 37
Substitution for Definite Integrals


Theorem (The Substitution Rule for Definite Integrals)
If g is continuous and f is continuous on the range of u = g (x), then
                                  b                                    g (b)
                                      f (g (x))g (x) dx =                      f (u) du.
                              a                                      g (a)



Why the change in the limits?
      The integral on the left happens in “x-land”
      The integral on the right happens in “u-land”, so the limits need to
      be u-values
      To get from x to u, apply g



 V63.0121.041, Calculus I (NYU)          Section 5.5 Integration by Substitution           December 13, 2010   22 / 37
Example
                   π
Compute                cos2 x sin x dx.
               0




V63.0121.041, Calculus I (NYU)      Section 5.5 Integration by Substitution   December 13, 2010   23 / 37
Example
                    π
Compute                 cos2 x sin x dx.
                0


Solution (Slow Way)

First compute the indefinite integral                        cos2 x sin x dx and then evaluate.
Let u = cos x . Then du = − sin x dx and

                          cos2 x sin x dx




 V63.0121.041, Calculus I (NYU)      Section 5.5 Integration by Substitution   December 13, 2010   23 / 37
Example
                    π
Compute                 cos2 x sin x dx.
                0


Solution (Slow Way)

First compute the indefinite integral                        cos2 x sin x dx and then evaluate.
Let u = cos x . Then du = − sin x dx and

                          cos2 x sin x dx




 V63.0121.041, Calculus I (NYU)      Section 5.5 Integration by Substitution   December 13, 2010   23 / 37
Example
                    π
Compute                 cos2 x sin x dx.
                0


Solution (Slow Way)

First compute the indefinite integral                        cos2 x sin x dx and then evaluate.
Let u = cos x . Then du = − sin x dx and

                          cos2 x sin x dx




 V63.0121.041, Calculus I (NYU)      Section 5.5 Integration by Substitution   December 13, 2010   23 / 37
Example
                    π
Compute                 cos2 x sin x dx.
                0


Solution (Slow Way)

First compute the indefinite integral                        cos2 x sin x dx and then evaluate.
Let u = cos x . Then du = − sin x dx and

                          cos2 x sin x dx = −            u 2 du




 V63.0121.041, Calculus I (NYU)      Section 5.5 Integration by Substitution   December 13, 2010   23 / 37
Example
                    π
Compute                 cos2 x sin x dx.
                0


Solution (Slow Way)

First compute the indefinite integral                        cos2 x sin x dx and then evaluate.
Let u = cos x . Then du = − sin x dx and

                          cos2 x sin x dx = −            u 2 du

                                            = − 3 u 3 + C = − 1 cos3 x + C .
                                                1
                                                              3




 V63.0121.041, Calculus I (NYU)      Section 5.5 Integration by Substitution   December 13, 2010   23 / 37
Example
                     π
Compute                  cos2 x sin x dx.
                 0


Solution (Slow Way)

First compute the indefinite integral                         cos2 x sin x dx and then evaluate.
Let u = cos x . Then du = − sin x dx and

                           cos2 x sin x dx = −            u 2 du

                                             = − 3 u 3 + C = − 1 cos3 x + C .
                                                 1
                                                               3

Therefore
                 π                                           π
                                        1
                     cos2 x sin x dx = − cos3 x
             0                          3                    0



 V63.0121.041, Calculus I (NYU)       Section 5.5 Integration by Substitution   December 13, 2010   23 / 37
Example
                     π
Compute                  cos2 x sin x dx.
                 0


Solution (Slow Way)

First compute the indefinite integral                         cos2 x sin x dx and then evaluate.
Let u = cos x . Then du = − sin x dx and

                           cos2 x sin x dx = −            u 2 du

                                             = − 3 u 3 + C = − 1 cos3 x + C .
                                                 1
                                                               3

Therefore
                 π                                           π
                                        1                                1
                     cos2 x sin x dx = − cos3 x                  =−        (−1)3 − 13
             0                          3                    0           3


 V63.0121.041, Calculus I (NYU)       Section 5.5 Integration by Substitution     December 13, 2010   23 / 37
Example
                     π
Compute                  cos2 x sin x dx.
                 0


Solution (Slow Way)

First compute the indefinite integral                         cos2 x sin x dx and then evaluate.
Let u = cos x . Then du = − sin x dx and

                           cos2 x sin x dx = −            u 2 du

                                             = − 3 u 3 + C = − 1 cos3 x + C .
                                                 1
                                                               3

Therefore
                 π                                           π
                                        1                                1             2
                     cos2 x sin x dx = − cos3 x                  =−        (−1)3 − 13 = .
             0                          3                    0           3             3


 V63.0121.041, Calculus I (NYU)       Section 5.5 Integration by Substitution    December 13, 2010   23 / 37
Definite-ly Quicker

Solution (Fast Way)
Do both the substitution and the evaluation at the same time. Let
u = cos x. Then du = − sin x dx, u(0) = 1 and u(π) = −1 . So
                        π
                            cos2 x sin x dx
                    0




 V63.0121.041, Calculus I (NYU)       Section 5.5 Integration by Substitution   December 13, 2010   24 / 37
Definite-ly Quicker

Solution (Fast Way)
Do both the substitution and the evaluation at the same time. Let
u = cos x. Then du = − sin x dx, u(0) = 1 and u(π) = −1 . So
                        π
                            cos2 x sin x dx
                    0




 V63.0121.041, Calculus I (NYU)       Section 5.5 Integration by Substitution   December 13, 2010   24 / 37
Definite-ly Quicker

Solution (Fast Way)
Do both the substitution and the evaluation at the same time. Let
u = cos x. Then du = − sin x dx, u(0) = 1 and u(π) = −1 . So
                        π
                            cos2 x sin x dx
                    0




 V63.0121.041, Calculus I (NYU)       Section 5.5 Integration by Substitution   December 13, 2010   24 / 37
Definite-ly Quicker

Solution (Fast Way)
Do both the substitution and the evaluation at the same time. Let
u = cos x. Then du = − sin x dx, u(0) = 1 and u(π) = −1 . So
                        π                               −1                     1
                            cos2 x sin x dx =                −u 2 du =              u 2 du
                    0                               1                          −1




 V63.0121.041, Calculus I (NYU)      Section 5.5 Integration by Substitution            December 13, 2010   24 / 37
Definite-ly Quicker

Solution (Fast Way)
Do both the substitution and the evaluation at the same time. Let
u = cos x. Then du = − sin x dx, u(0) = 1 and u(π) = −1 . So
                        π                               −1                     1
                            cos2 x sin x dx =                −u 2 du =              u 2 du
                    0                               1                          −1
                                                             1
                                                  1 3                 1            2
                                             =      u             =     1 − (−1) =
                                                  3          −1       3            3




 V63.0121.041, Calculus I (NYU)      Section 5.5 Integration by Substitution            December 13, 2010   24 / 37
Definite-ly Quicker

Solution (Fast Way)
Do both the substitution and the evaluation at the same time. Let
u = cos x. Then du = − sin x dx, u(0) = 1 and u(π) = −1 . So
                        π                               −1                     1
                            cos2 x sin x dx =                −u 2 du =              u 2 du
                    0                               1                          −1
                                                             1
                                                  1 3                 1            2
                                             =      u             =     1 − (−1) =
                                                  3          −1       3            3


      The advantage to the “fast way” is that you completely transform the
      integral into something simpler and don’t have to go back to the
      original variable (x).


 V63.0121.041, Calculus I (NYU)      Section 5.5 Integration by Substitution            December 13, 2010   24 / 37
Definite-ly Quicker

Solution (Fast Way)
Do both the substitution and the evaluation at the same time. Let
u = cos x. Then du = − sin x dx, u(0) = 1 and u(π) = −1 . So
                        π                               −1                     1
                            cos2 x sin x dx =                −u 2 du =              u 2 du
                    0                               1                          −1
                                                             1
                                                  1 3                 1            2
                                             =      u             =     1 − (−1) =
                                                  3          −1       3            3


      The advantage to the “fast way” is that you completely transform the
      integral into something simpler and don’t have to go back to the
      original variable (x).
      But the slow way is just as reliable.

 V63.0121.041, Calculus I (NYU)      Section 5.5 Integration by Substitution            December 13, 2010   24 / 37
An exponential example
Example
               √
          ln          8
Find          √           e 2x   e 2x + 1 dx
         ln       3




 V63.0121.041, Calculus I (NYU)          Section 5.5 Integration by Substitution   December 13, 2010   25 / 37
An exponential example
Example
               √
          ln          8
Find          √           e 2x   e 2x + 1 dx
         ln       3

Solution
Let u = e 2x , so du = 2e 2x dx. We have
                                       √
                                  ln          8                                           8√
                                                                                 1
                                      √           e 2x   e 2x + 1 dx =                         u + 1 du
                                 ln       3                                      2    3




 V63.0121.041, Calculus I (NYU)                      Section 5.5 Integration by Substitution         December 13, 2010   25 / 37
About those limits




Since
                                                    √                 √   2
                                           e 2(ln       3)
                                                             = e ln       3
                                                                              = e ln 3 = 3

we have                         √
                           ln          8                                               8√
                                                                               1
                               √           e 2x     e 2x + 1 dx =                           u + 1 du
                          ln       3                                           2   3




 V63.0121.041, Calculus I (NYU)                Section 5.5 Integration by Substitution            December 13, 2010   26 / 37
An exponential example
Example
               √
          ln          8
Find          √           e 2x       e 2x + 1 dx
         ln       3

Solution
Let u = e 2x , so du = 2e 2x dx. We have
                                           √
                                      ln          8                                            8√
                                                                                      1
                                          √           e 2x   e 2x + 1 dx =                          u + 1 du
                                     ln       3                                       2    3

Now let y = u + 1, dy = du. So
                                     8√                                     9                           9
                          1                                        1            √              1
                                           u + 1 du =                               y dy =                  y 1/2 dy
                          2      3                                 2    4                      2    4
                                                                                      9
                                                               1 2                         1           19
                                                              = · y 3/2                   = (27 − 8) =
                                                               2 3                    4    3           3
 V63.0121.041, Calculus I (NYU)                          Section 5.5 Integration by Substitution                December 13, 2010   27 / 37
About those fractional powers



We have

                                      93/2 = (91/2 )3 = 33 = 27
                                      43/2 = (41/2 )3 = 23 = 8

so
                             9                                 9
                    1                         1 2 3/2                  1            19
                                 y 1/2 dy =    · y                 =     (27 − 8) =
                    2    4                    2 3              4       3            3




 V63.0121.041, Calculus I (NYU)        Section 5.5 Integration by Substitution    December 13, 2010   28 / 37
An exponential example
Example
               √
          ln          8
Find          √           e 2x       e 2x + 1 dx
         ln       3

Solution
Let u = e 2x , so du = 2e 2x dx. We have
                                           √
                                      ln          8                                            8√
                                                                                      1
                                          √           e 2x   e 2x + 1 dx =                          u + 1 du
                                     ln       3                                       2    3

Now let y = u + 1, dy = du. So
                                     8√                                     9                           9
                          1                                        1            √              1
                                           u + 1 du =                               y dy =                  y 1/2 dy
                          2      3                                 2    4                      2    4
                                                                                      9
                                                               1 2                         1           19
                                                              = · y 3/2                   = (27 − 8) =
                                                               2 3                    4    3           3
 V63.0121.041, Calculus I (NYU)                          Section 5.5 Integration by Substitution                December 13, 2010   29 / 37
Another way to skin that cat

Example
               √
          ln          8
Find          √           e 2x   e 2x + 1 dx
         ln       3


Solution
Let u = e 2x + 1,




 V63.0121.041, Calculus I (NYU)          Section 5.5 Integration by Substitution   December 13, 2010   30 / 37
Another way to skin that cat

Example
               √
          ln          8
Find          √           e 2x   e 2x + 1 dx
         ln       3


Solution
Let u = e 2x + 1,so that du = 2e 2x dx.




 V63.0121.041, Calculus I (NYU)          Section 5.5 Integration by Substitution   December 13, 2010   30 / 37
Another way to skin that cat

Example
               √
          ln          8
Find          √           e 2x   e 2x + 1 dx
         ln       3


Solution
Let u = e 2x + 1,so that du = 2e 2x dx. Then
                                       √
                                  ln          8                                           9√
                                                                                 1
                                      √           e 2x   e 2x + 1 dx =                         u du
                                 ln       3                                      2    4




 V63.0121.041, Calculus I (NYU)                      Section 5.5 Integration by Substitution          December 13, 2010   30 / 37
Another way to skin that cat

Example
               √
          ln          8
Find          √           e 2x   e 2x + 1 dx
         ln       3


Solution
Let u = e 2x + 1,so that du = 2e 2x dx. Then
                                       √
                                  ln          8                                           9√
                                                                                 1
                                      √           e 2x   e 2x + 1 dx =                             u du
                                 ln       3                                      2    4
                                                                                               9
                                                                             1
                                                                            = u 3/2
                                                                             3                 4




 V63.0121.041, Calculus I (NYU)                      Section 5.5 Integration by Substitution              December 13, 2010   30 / 37
Another way to skin that cat

Example
               √
          ln          8
Find          √           e 2x   e 2x + 1 dx
         ln       3


Solution
Let u = e 2x + 1,so that du = 2e 2x dx. Then
                                       √
                                  ln          8                                           9√
                                                                                 1
                                      √           e 2x   e 2x + 1 dx =                         u du
                                 ln       3                                      2    4
                                                                             1 3/2 9
                                                                            = u
                                                                             3     4
                                                                             1           19
                                                                            = (27 − 8) =
                                                                             3           3


 V63.0121.041, Calculus I (NYU)                      Section 5.5 Integration by Substitution          December 13, 2010   30 / 37
A third skinned cat


Example
               √
          ln          8
Find          √           e 2x   e 2x + 1 dx
         ln       3


Solution
Let u =               e 2x + 1, so that

                                  u 2 = e 2x + 1




 V63.0121.041, Calculus I (NYU)          Section 5.5 Integration by Substitution   December 13, 2010   31 / 37
A third skinned cat


Example
               √
          ln          8
Find          √           e 2x   e 2x + 1 dx
         ln       3


Solution
Let u =               e 2x + 1, so that

                                  u 2 = e 2x + 1 =⇒ 2u du = 2e 2x dx




 V63.0121.041, Calculus I (NYU)          Section 5.5 Integration by Substitution   December 13, 2010   31 / 37
A third skinned cat


Example
               √
          ln          8
Find          √           e 2x   e 2x + 1 dx
         ln       3


Solution
Let u =               e 2x + 1, so that

                                  u 2 = e 2x + 1 =⇒ 2u du = 2e 2x dx

Thus                                     √
                                    ln          8              3                          3
                                                                                  1 3             19
                                        √            =             u · u du =       u         =
                                   ln       3              2                      3       2       3



 V63.0121.041, Calculus I (NYU)                     Section 5.5 Integration by Substitution            December 13, 2010   31 / 37
A Trigonometric Example



Example
Find
                                  3π/2
                                                      θ                θ
                                          cot5              sec2                dθ.
                                  π                   6                6




 V63.0121.041, Calculus I (NYU)       Section 5.5 Integration by Substitution         December 13, 2010   32 / 37
A Trigonometric Example



Example
Find
                                  3π/2
                                                      θ                θ
                                          cot5              sec2                dθ.
                                  π                   6                6

Before we dive in, think about:
       What “easy” substitutions might help?
       Which of the trig functions suggests a substitution?




 V63.0121.041, Calculus I (NYU)       Section 5.5 Integration by Substitution         December 13, 2010   32 / 37
Solution
       θ           1
Let ϕ = . Then dϕ = dθ.
       6           6
                3π/2                                                    π/4
                                  θ              θ
                       cot5           sec2             dθ = 6                   cot5 ϕ sec2 ϕ dϕ
               π                  6              6                    π/6
                                                                       π/4
                                                                                sec2 ϕ dϕ
                                                             =6
                                                                      π/6         tan5 ϕ




 V63.0121.041, Calculus I (NYU)       Section 5.5 Integration by Substitution          December 13, 2010   33 / 37
Solution
       θ           1
Let ϕ = . Then dϕ = dθ.
       6           6
                3π/2                                                    π/4
                                  θ              θ
                          cot5        sec2             dθ = 6                   cot5 ϕ sec2 ϕ dϕ
                π                 6              6                     π/6
                                                                        π/4
                                                                                sec2 ϕ dϕ
                                                             =6
                                                                       π/6        tan5 ϕ

Now let u = tan ϕ. So du = sec2 ϕ dϕ, and
                     π/4                           1
                           sec2 ϕ dϕ                       −5
            6                        =6            √ u          du
                    π/6      tan5 ϕ              1/ 3
                                                                   1
                                                   1                             3
                                        =6        − u −4             √
                                                                             =     [9 − 1] = 12.
                                                   4               1/ 3          2



 V63.0121.041, Calculus I (NYU)       Section 5.5 Integration by Substitution          December 13, 2010   33 / 37
Graphs
                       3π/2                                                            π/4
                                         θ                θ
                              cot   5
                                               sec   2
                                                                 dθ                          6 cot5 ϕ sec2 ϕ dϕ
                     π                   6                6                           π/6

                 y                                                                y




                                                                       θ                              ϕ
                                        3π       π                                    ππ
                                         2                                            64
The areas of these two regions are the same.


 V63.0121.041, Calculus I (NYU)         Section 5.5 Integration by Substitution              December 13, 2010   35 / 37
Graphs
                       π/4                                                       1
                                                                                          −5
                             6 cot5 ϕ sec2 ϕ dϕ                                    √ 6u        du
                     π/6                                                         1/ 3

                 y                                                           y




                                   ϕ                                                 u
                      ππ                                    1 1
                      64                                   √
                                                             3
                      The areas of these two regions are the same.


 V63.0121.041, Calculus I (NYU)    Section 5.5 Integration by Substitution           December 13, 2010   36 / 37
Summary


     If F is an antiderivative for f , then:

                                     f (g (x))g (x) dx = F (g (x))

     If F is an antiderivative for f , which is continuous on the range of g ,
     then:
                   b                               g (b)
                       f (g (x))g (x) dx =                 f (u) du = F (g (b)) − F (g (a))
               a                                 g (a)

     Antidifferentiation in general and substitution in particular is a
     “nonlinear” problem that needs practice, intuition, and perserverance
     The whole antidifferentiation story is in Chapter 6


V63.0121.041, Calculus I (NYU)     Section 5.5 Integration by Substitution   December 13, 2010   37 / 37

More Related Content

What's hot

Formulas
FormulasFormulas
FormulasZHERILL
 
Differentiation
DifferentiationDifferentiation
Differentiationtimschmitz
 
Murphy: Machine learning A probabilistic perspective: Ch.9
Murphy: Machine learning A probabilistic perspective: Ch.9Murphy: Machine learning A probabilistic perspective: Ch.9
Murphy: Machine learning A probabilistic perspective: Ch.9Daisuke Yoneoka
 
Lesson 8: Basic Differentiation Rules (Section 21 slides)
Lesson 8: Basic Differentiation Rules (Section 21 slides)Lesson 8: Basic Differentiation Rules (Section 21 slides)
Lesson 8: Basic Differentiation Rules (Section 21 slides)Matthew Leingang
 
Continuity of functions by graph (exercises with detailed solutions)
Continuity of functions by graph   (exercises with detailed solutions)Continuity of functions by graph   (exercises with detailed solutions)
Continuity of functions by graph (exercises with detailed solutions)Tarun Gehlot
 
Deep generative model.pdf
Deep generative model.pdfDeep generative model.pdf
Deep generative model.pdfHyungjoo Cho
 
Review 1 -_limits-_continuity_(pcalc+_to_ap_calc)
Review 1 -_limits-_continuity_(pcalc+_to_ap_calc)Review 1 -_limits-_continuity_(pcalc+_to_ap_calc)
Review 1 -_limits-_continuity_(pcalc+_to_ap_calc)Ron Eick
 
Interpolation with unequal interval
Interpolation with unequal intervalInterpolation with unequal interval
Interpolation with unequal intervalDr. Nirav Vyas
 
Lesson 2: Limits and Limit Laws
Lesson 2: Limits and Limit LawsLesson 2: Limits and Limit Laws
Lesson 2: Limits and Limit LawsMatthew Leingang
 
Linear approximations and_differentials
Linear approximations and_differentialsLinear approximations and_differentials
Linear approximations and_differentialsTarun Gehlot
 
The Application of Derivatives
The Application of DerivativesThe Application of Derivatives
The Application of Derivativesdivaprincess09
 
CHAIN RULE AND IMPLICIT FUNCTION
CHAIN RULE AND IMPLICIT FUNCTIONCHAIN RULE AND IMPLICIT FUNCTION
CHAIN RULE AND IMPLICIT FUNCTIONNikhil Pandit
 
Lesson 8: Basic Differentation Rules (slides)
Lesson 8: Basic Differentation Rules (slides)Lesson 8: Basic Differentation Rules (slides)
Lesson 8: Basic Differentation Rules (slides)Matthew Leingang
 
Harmonic Analysis and Deep Learning
Harmonic Analysis and Deep LearningHarmonic Analysis and Deep Learning
Harmonic Analysis and Deep LearningSungbin Lim
 

What's hot (19)

Formulas
FormulasFormulas
Formulas
 
Differentiation
DifferentiationDifferentiation
Differentiation
 
Imc2017 day1-solutions
Imc2017 day1-solutionsImc2017 day1-solutions
Imc2017 day1-solutions
 
Murphy: Machine learning A probabilistic perspective: Ch.9
Murphy: Machine learning A probabilistic perspective: Ch.9Murphy: Machine learning A probabilistic perspective: Ch.9
Murphy: Machine learning A probabilistic perspective: Ch.9
 
Lesson 8: Basic Differentiation Rules (Section 21 slides)
Lesson 8: Basic Differentiation Rules (Section 21 slides)Lesson 8: Basic Differentiation Rules (Section 21 slides)
Lesson 8: Basic Differentiation Rules (Section 21 slides)
 
Limits and derivatives
Limits and derivativesLimits and derivatives
Limits and derivatives
 
Continuity of functions by graph (exercises with detailed solutions)
Continuity of functions by graph   (exercises with detailed solutions)Continuity of functions by graph   (exercises with detailed solutions)
Continuity of functions by graph (exercises with detailed solutions)
 
Deep generative model.pdf
Deep generative model.pdfDeep generative model.pdf
Deep generative model.pdf
 
Rules of derivative
Rules of derivativeRules of derivative
Rules of derivative
 
Review 1 -_limits-_continuity_(pcalc+_to_ap_calc)
Review 1 -_limits-_continuity_(pcalc+_to_ap_calc)Review 1 -_limits-_continuity_(pcalc+_to_ap_calc)
Review 1 -_limits-_continuity_(pcalc+_to_ap_calc)
 
Interpolation with unequal interval
Interpolation with unequal intervalInterpolation with unequal interval
Interpolation with unequal interval
 
Lesson 5: Continuity
Lesson 5: ContinuityLesson 5: Continuity
Lesson 5: Continuity
 
Lesson 2: Limits and Limit Laws
Lesson 2: Limits and Limit LawsLesson 2: Limits and Limit Laws
Lesson 2: Limits and Limit Laws
 
Matrix calculus
Matrix calculusMatrix calculus
Matrix calculus
 
Linear approximations and_differentials
Linear approximations and_differentialsLinear approximations and_differentials
Linear approximations and_differentials
 
The Application of Derivatives
The Application of DerivativesThe Application of Derivatives
The Application of Derivatives
 
CHAIN RULE AND IMPLICIT FUNCTION
CHAIN RULE AND IMPLICIT FUNCTIONCHAIN RULE AND IMPLICIT FUNCTION
CHAIN RULE AND IMPLICIT FUNCTION
 
Lesson 8: Basic Differentation Rules (slides)
Lesson 8: Basic Differentation Rules (slides)Lesson 8: Basic Differentation Rules (slides)
Lesson 8: Basic Differentation Rules (slides)
 
Harmonic Analysis and Deep Learning
Harmonic Analysis and Deep LearningHarmonic Analysis and Deep Learning
Harmonic Analysis and Deep Learning
 

Viewers also liked

Lesson 27: Integration by Substitution (Section 041 handout)
Lesson 27: Integration by Substitution (Section 041 handout)Lesson 27: Integration by Substitution (Section 041 handout)
Lesson 27: Integration by Substitution (Section 041 handout)Matthew Leingang
 
Lesson 26: The Fundamental Theorem of Calculus (Section 041 slides)
Lesson 26: The Fundamental Theorem of Calculus (Section 041 slides)Lesson 26: The Fundamental Theorem of Calculus (Section 041 slides)
Lesson 26: The Fundamental Theorem of Calculus (Section 041 slides)Matthew Leingang
 
Lesson 15: Exponential Growth and Decay (Section 021 slides)
Lesson 15: Exponential Growth and Decay (Section 021 slides)Lesson 15: Exponential Growth and Decay (Section 021 slides)
Lesson 15: Exponential Growth and Decay (Section 021 slides)Matthew Leingang
 
Lesson 24: Areas, Distances, the Integral (Section 021 slides)
Lesson 24: Areas, Distances, the Integral (Section 021 slides)Lesson 24: Areas, Distances, the Integral (Section 021 slides)
Lesson 24: Areas, Distances, the Integral (Section 021 slides)Matthew Leingang
 
Lesson 26: The Fundamental Theorem of Calculus (Section 021 slides)
Lesson 26: The Fundamental Theorem of Calculus (Section 021 slides)Lesson 26: The Fundamental Theorem of Calculus (Section 021 slides)
Lesson 26: The Fundamental Theorem of Calculus (Section 021 slides)Matthew Leingang
 
Lesson 25: Evaluating Definite Integrals (Section 041 slides)
Lesson 25: Evaluating Definite Integrals (Section 041 slides)Lesson 25: Evaluating Definite Integrals (Section 041 slides)
Lesson 25: Evaluating Definite Integrals (Section 041 slides)Matthew Leingang
 
Lesson 12: Linear Approximations and Differentials (slides)
Lesson 12: Linear Approximations and Differentials (slides)Lesson 12: Linear Approximations and Differentials (slides)
Lesson 12: Linear Approximations and Differentials (slides)Matthew Leingang
 
Lesson 9: The Product and Quotient Rules (slides)
Lesson 9: The Product and Quotient Rules (slides)Lesson 9: The Product and Quotient Rules (slides)
Lesson 9: The Product and Quotient Rules (slides)Matthew Leingang
 
Lesson 2: A Catalog of Essential Functions (slides)
Lesson 2: A Catalog of Essential Functions (slides)Lesson 2: A Catalog of Essential Functions (slides)
Lesson 2: A Catalog of Essential Functions (slides)Matthew Leingang
 
Lesson 1: Functions and their representations (slides)
Lesson 1: Functions and their representations (slides)Lesson 1: Functions and their representations (slides)
Lesson 1: Functions and their representations (slides)Matthew Leingang
 
Lesson 24: Areas, Distances, the Integral (Section 041 slides)
Lesson 24: Areas, Distances, the Integral (Section 041 slides)Lesson 24: Areas, Distances, the Integral (Section 041 slides)
Lesson 24: Areas, Distances, the Integral (Section 041 slides)Matthew Leingang
 

Viewers also liked (14)

Lesson 27: Integration by Substitution (Section 041 handout)
Lesson 27: Integration by Substitution (Section 041 handout)Lesson 27: Integration by Substitution (Section 041 handout)
Lesson 27: Integration by Substitution (Section 041 handout)
 
Lesson 26: The Fundamental Theorem of Calculus (Section 041 slides)
Lesson 26: The Fundamental Theorem of Calculus (Section 041 slides)Lesson 26: The Fundamental Theorem of Calculus (Section 041 slides)
Lesson 26: The Fundamental Theorem of Calculus (Section 041 slides)
 
Lesson 15: Exponential Growth and Decay (Section 021 slides)
Lesson 15: Exponential Growth and Decay (Section 021 slides)Lesson 15: Exponential Growth and Decay (Section 021 slides)
Lesson 15: Exponential Growth and Decay (Section 021 slides)
 
Lesson 24: Areas, Distances, the Integral (Section 021 slides)
Lesson 24: Areas, Distances, the Integral (Section 021 slides)Lesson 24: Areas, Distances, the Integral (Section 021 slides)
Lesson 24: Areas, Distances, the Integral (Section 021 slides)
 
Lesson 26: The Fundamental Theorem of Calculus (Section 021 slides)
Lesson 26: The Fundamental Theorem of Calculus (Section 021 slides)Lesson 26: The Fundamental Theorem of Calculus (Section 021 slides)
Lesson 26: The Fundamental Theorem of Calculus (Section 021 slides)
 
Lesson 25: Evaluating Definite Integrals (Section 041 slides)
Lesson 25: Evaluating Definite Integrals (Section 041 slides)Lesson 25: Evaluating Definite Integrals (Section 041 slides)
Lesson 25: Evaluating Definite Integrals (Section 041 slides)
 
Lesson 12: Linear Approximations and Differentials (slides)
Lesson 12: Linear Approximations and Differentials (slides)Lesson 12: Linear Approximations and Differentials (slides)
Lesson 12: Linear Approximations and Differentials (slides)
 
Lesson 9: The Product and Quotient Rules (slides)
Lesson 9: The Product and Quotient Rules (slides)Lesson 9: The Product and Quotient Rules (slides)
Lesson 9: The Product and Quotient Rules (slides)
 
Lesson 2: A Catalog of Essential Functions (slides)
Lesson 2: A Catalog of Essential Functions (slides)Lesson 2: A Catalog of Essential Functions (slides)
Lesson 2: A Catalog of Essential Functions (slides)
 
Lesson 1: Functions and their representations (slides)
Lesson 1: Functions and their representations (slides)Lesson 1: Functions and their representations (slides)
Lesson 1: Functions and their representations (slides)
 
Lesson 24: Areas, Distances, the Integral (Section 041 slides)
Lesson 24: Areas, Distances, the Integral (Section 041 slides)Lesson 24: Areas, Distances, the Integral (Section 041 slides)
Lesson 24: Areas, Distances, the Integral (Section 041 slides)
 
Set Theory QA 3
Set Theory QA 3Set Theory QA 3
Set Theory QA 3
 
Permutation and Combination 2
Permutation and Combination 2Permutation and Combination 2
Permutation and Combination 2
 
Wave Motion QA 3
Wave Motion QA 3Wave Motion QA 3
Wave Motion QA 3
 

Similar to Lesson 27: Integration by Substitution (Section 041 slides)

Lesson 27: Integration by Substitution (Section 021 handout)
Lesson 27: Integration by Substitution (Section 021 handout)Lesson 27: Integration by Substitution (Section 021 handout)
Lesson 27: Integration by Substitution (Section 021 handout)Matthew Leingang
 
Lesson 28: Integration by Subsitution
Lesson 28: Integration by SubsitutionLesson 28: Integration by Subsitution
Lesson 28: Integration by SubsitutionMel Anthony Pepito
 
Lesson 28: Integration by Subsitution
Lesson 28: Integration by SubsitutionLesson 28: Integration by Subsitution
Lesson 28: Integration by SubsitutionMatthew Leingang
 
Lesson 29: Integration by Substition
Lesson 29: Integration by SubstitionLesson 29: Integration by Substition
Lesson 29: Integration by SubstitionMatthew Leingang
 
Lesson 29: Integration by Substition (worksheet solutions)
Lesson 29: Integration by Substition (worksheet solutions)Lesson 29: Integration by Substition (worksheet solutions)
Lesson 29: Integration by Substition (worksheet solutions)Matthew Leingang
 
Lesson 27: Integration by Substitution (slides)
Lesson 27: Integration by Substitution (slides)Lesson 27: Integration by Substitution (slides)
Lesson 27: Integration by Substitution (slides)Mel Anthony Pepito
 
Lesson 26: The Fundamental Theorem of Calculus (Section 041 slides)
Lesson 26: The Fundamental Theorem of Calculus (Section 041 slides)Lesson 26: The Fundamental Theorem of Calculus (Section 041 slides)
Lesson 26: The Fundamental Theorem of Calculus (Section 041 slides)Mel Anthony Pepito
 
Lesson 26: The Fundamental Theorem of Calculus (Section 021 slides)
Lesson 26: The Fundamental Theorem of Calculus (Section 021 slides)Lesson 26: The Fundamental Theorem of Calculus (Section 021 slides)
Lesson 26: The Fundamental Theorem of Calculus (Section 021 slides)Mel Anthony Pepito
 
Lesson 27: The Fundamental Theorem of Calculus
Lesson 27: The Fundamental Theorem of CalculusLesson 27: The Fundamental Theorem of Calculus
Lesson 27: The Fundamental Theorem of CalculusMatthew Leingang
 
Lesson 27: The Fundamental Theorem of Calculus
Lesson 27: The Fundamental Theorem of Calculus Lesson 27: The Fundamental Theorem of Calculus
Lesson 27: The Fundamental Theorem of Calculus Mel Anthony Pepito
 
Lesson 27: Integration by Substitution (Section 4 version)
Lesson 27: Integration by Substitution (Section 4 version)Lesson 27: Integration by Substitution (Section 4 version)
Lesson 27: Integration by Substitution (Section 4 version)Matthew Leingang
 
Lesson 26: Integration by Substitution (slides)
Lesson 26: Integration by Substitution (slides)Lesson 26: Integration by Substitution (slides)
Lesson 26: Integration by Substitution (slides)Matthew Leingang
 
Lesson 26: Integration by Substitution (handout)
Lesson 26: Integration by Substitution (handout)Lesson 26: Integration by Substitution (handout)
Lesson 26: Integration by Substitution (handout)Matthew Leingang
 
1.1_The_Definite_Integral.pdf odjoqwddoio
1.1_The_Definite_Integral.pdf odjoqwddoio1.1_The_Definite_Integral.pdf odjoqwddoio
1.1_The_Definite_Integral.pdf odjoqwddoioNoorYassinHJamel
 
Lesson 23: Antiderivatives (Section 021 slides)
Lesson 23: Antiderivatives (Section 021 slides)Lesson 23: Antiderivatives (Section 021 slides)
Lesson 23: Antiderivatives (Section 021 slides)Mel Anthony Pepito
 
Lesson 27: Integration by Substitution (Section 10 version)
Lesson 27: Integration by Substitution (Section 10 version)Lesson 27: Integration by Substitution (Section 10 version)
Lesson 27: Integration by Substitution (Section 10 version)Matthew Leingang
 
Lesson 28: The Fundamental Theorem of Calculus
Lesson 28: The Fundamental Theorem of CalculusLesson 28: The Fundamental Theorem of Calculus
Lesson 28: The Fundamental Theorem of CalculusMatthew Leingang
 
Lesson 28: The Fundamental Theorem of Calculus
Lesson 28: The Fundamental Theorem of CalculusLesson 28: The Fundamental Theorem of Calculus
Lesson 28: The Fundamental Theorem of CalculusMatthew Leingang
 
Lesson 5: Continuity (Section 21 slides)
Lesson 5: Continuity (Section 21 slides)Lesson 5: Continuity (Section 21 slides)
Lesson 5: Continuity (Section 21 slides)Matthew Leingang
 
Lesson 5: Continuity (Section 21 slides)
Lesson 5: Continuity (Section 21 slides)Lesson 5: Continuity (Section 21 slides)
Lesson 5: Continuity (Section 21 slides)Mel Anthony Pepito
 

Similar to Lesson 27: Integration by Substitution (Section 041 slides) (20)

Lesson 27: Integration by Substitution (Section 021 handout)
Lesson 27: Integration by Substitution (Section 021 handout)Lesson 27: Integration by Substitution (Section 021 handout)
Lesson 27: Integration by Substitution (Section 021 handout)
 
Lesson 28: Integration by Subsitution
Lesson 28: Integration by SubsitutionLesson 28: Integration by Subsitution
Lesson 28: Integration by Subsitution
 
Lesson 28: Integration by Subsitution
Lesson 28: Integration by SubsitutionLesson 28: Integration by Subsitution
Lesson 28: Integration by Subsitution
 
Lesson 29: Integration by Substition
Lesson 29: Integration by SubstitionLesson 29: Integration by Substition
Lesson 29: Integration by Substition
 
Lesson 29: Integration by Substition (worksheet solutions)
Lesson 29: Integration by Substition (worksheet solutions)Lesson 29: Integration by Substition (worksheet solutions)
Lesson 29: Integration by Substition (worksheet solutions)
 
Lesson 27: Integration by Substitution (slides)
Lesson 27: Integration by Substitution (slides)Lesson 27: Integration by Substitution (slides)
Lesson 27: Integration by Substitution (slides)
 
Lesson 26: The Fundamental Theorem of Calculus (Section 041 slides)
Lesson 26: The Fundamental Theorem of Calculus (Section 041 slides)Lesson 26: The Fundamental Theorem of Calculus (Section 041 slides)
Lesson 26: The Fundamental Theorem of Calculus (Section 041 slides)
 
Lesson 26: The Fundamental Theorem of Calculus (Section 021 slides)
Lesson 26: The Fundamental Theorem of Calculus (Section 021 slides)Lesson 26: The Fundamental Theorem of Calculus (Section 021 slides)
Lesson 26: The Fundamental Theorem of Calculus (Section 021 slides)
 
Lesson 27: The Fundamental Theorem of Calculus
Lesson 27: The Fundamental Theorem of CalculusLesson 27: The Fundamental Theorem of Calculus
Lesson 27: The Fundamental Theorem of Calculus
 
Lesson 27: The Fundamental Theorem of Calculus
Lesson 27: The Fundamental Theorem of Calculus Lesson 27: The Fundamental Theorem of Calculus
Lesson 27: The Fundamental Theorem of Calculus
 
Lesson 27: Integration by Substitution (Section 4 version)
Lesson 27: Integration by Substitution (Section 4 version)Lesson 27: Integration by Substitution (Section 4 version)
Lesson 27: Integration by Substitution (Section 4 version)
 
Lesson 26: Integration by Substitution (slides)
Lesson 26: Integration by Substitution (slides)Lesson 26: Integration by Substitution (slides)
Lesson 26: Integration by Substitution (slides)
 
Lesson 26: Integration by Substitution (handout)
Lesson 26: Integration by Substitution (handout)Lesson 26: Integration by Substitution (handout)
Lesson 26: Integration by Substitution (handout)
 
1.1_The_Definite_Integral.pdf odjoqwddoio
1.1_The_Definite_Integral.pdf odjoqwddoio1.1_The_Definite_Integral.pdf odjoqwddoio
1.1_The_Definite_Integral.pdf odjoqwddoio
 
Lesson 23: Antiderivatives (Section 021 slides)
Lesson 23: Antiderivatives (Section 021 slides)Lesson 23: Antiderivatives (Section 021 slides)
Lesson 23: Antiderivatives (Section 021 slides)
 
Lesson 27: Integration by Substitution (Section 10 version)
Lesson 27: Integration by Substitution (Section 10 version)Lesson 27: Integration by Substitution (Section 10 version)
Lesson 27: Integration by Substitution (Section 10 version)
 
Lesson 28: The Fundamental Theorem of Calculus
Lesson 28: The Fundamental Theorem of CalculusLesson 28: The Fundamental Theorem of Calculus
Lesson 28: The Fundamental Theorem of Calculus
 
Lesson 28: The Fundamental Theorem of Calculus
Lesson 28: The Fundamental Theorem of CalculusLesson 28: The Fundamental Theorem of Calculus
Lesson 28: The Fundamental Theorem of Calculus
 
Lesson 5: Continuity (Section 21 slides)
Lesson 5: Continuity (Section 21 slides)Lesson 5: Continuity (Section 21 slides)
Lesson 5: Continuity (Section 21 slides)
 
Lesson 5: Continuity (Section 21 slides)
Lesson 5: Continuity (Section 21 slides)Lesson 5: Continuity (Section 21 slides)
Lesson 5: Continuity (Section 21 slides)
 

More from Matthew Leingang

Streamlining assessment, feedback, and archival with auto-multiple-choice
Streamlining assessment, feedback, and archival with auto-multiple-choiceStreamlining assessment, feedback, and archival with auto-multiple-choice
Streamlining assessment, feedback, and archival with auto-multiple-choiceMatthew Leingang
 
Electronic Grading of Paper Assessments
Electronic Grading of Paper AssessmentsElectronic Grading of Paper Assessments
Electronic Grading of Paper AssessmentsMatthew Leingang
 
Lesson 26: The Fundamental Theorem of Calculus (slides)
Lesson 26: The Fundamental Theorem of Calculus (slides)Lesson 26: The Fundamental Theorem of Calculus (slides)
Lesson 26: The Fundamental Theorem of Calculus (slides)Matthew Leingang
 
Lesson 27: Integration by Substitution (handout)
Lesson 27: Integration by Substitution (handout)Lesson 27: Integration by Substitution (handout)
Lesson 27: Integration by Substitution (handout)Matthew Leingang
 
Lesson 26: The Fundamental Theorem of Calculus (handout)
Lesson 26: The Fundamental Theorem of Calculus (handout)Lesson 26: The Fundamental Theorem of Calculus (handout)
Lesson 26: The Fundamental Theorem of Calculus (handout)Matthew Leingang
 
Lesson 25: Evaluating Definite Integrals (slides)
Lesson 25: Evaluating Definite Integrals (slides)Lesson 25: Evaluating Definite Integrals (slides)
Lesson 25: Evaluating Definite Integrals (slides)Matthew Leingang
 
Lesson 25: Evaluating Definite Integrals (handout)
Lesson 25: Evaluating Definite Integrals (handout)Lesson 25: Evaluating Definite Integrals (handout)
Lesson 25: Evaluating Definite Integrals (handout)Matthew Leingang
 
Lesson 24: Areas and Distances, The Definite Integral (handout)
Lesson 24: Areas and Distances, The Definite Integral (handout)Lesson 24: Areas and Distances, The Definite Integral (handout)
Lesson 24: Areas and Distances, The Definite Integral (handout)Matthew Leingang
 
Lesson 24: Areas and Distances, The Definite Integral (slides)
Lesson 24: Areas and Distances, The Definite Integral (slides)Lesson 24: Areas and Distances, The Definite Integral (slides)
Lesson 24: Areas and Distances, The Definite Integral (slides)Matthew Leingang
 
Lesson 23: Antiderivatives (slides)
Lesson 23: Antiderivatives (slides)Lesson 23: Antiderivatives (slides)
Lesson 23: Antiderivatives (slides)Matthew Leingang
 
Lesson 23: Antiderivatives (slides)
Lesson 23: Antiderivatives (slides)Lesson 23: Antiderivatives (slides)
Lesson 23: Antiderivatives (slides)Matthew Leingang
 
Lesson 22: Optimization Problems (slides)
Lesson 22: Optimization Problems (slides)Lesson 22: Optimization Problems (slides)
Lesson 22: Optimization Problems (slides)Matthew Leingang
 
Lesson 22: Optimization Problems (handout)
Lesson 22: Optimization Problems (handout)Lesson 22: Optimization Problems (handout)
Lesson 22: Optimization Problems (handout)Matthew Leingang
 
Lesson 21: Curve Sketching (handout)
Lesson 21: Curve Sketching (handout)Lesson 21: Curve Sketching (handout)
Lesson 21: Curve Sketching (handout)Matthew Leingang
 
Lesson 20: Derivatives and the Shapes of Curves (slides)
Lesson 20: Derivatives and the Shapes of Curves (slides)Lesson 20: Derivatives and the Shapes of Curves (slides)
Lesson 20: Derivatives and the Shapes of Curves (slides)Matthew Leingang
 
Lesson 20: Derivatives and the Shapes of Curves (handout)
Lesson 20: Derivatives and the Shapes of Curves (handout)Lesson 20: Derivatives and the Shapes of Curves (handout)
Lesson 20: Derivatives and the Shapes of Curves (handout)Matthew Leingang
 
Lesson 19: The Mean Value Theorem (slides)
Lesson 19: The Mean Value Theorem (slides)Lesson 19: The Mean Value Theorem (slides)
Lesson 19: The Mean Value Theorem (slides)Matthew Leingang
 
Lesson 17: Indeterminate forms and l'Hôpital's Rule (slides)
Lesson 17: Indeterminate forms and l'Hôpital's Rule (slides)Lesson 17: Indeterminate forms and l'Hôpital's Rule (slides)
Lesson 17: Indeterminate forms and l'Hôpital's Rule (slides)Matthew Leingang
 
Lesson 18: Maximum and Minimum Values (handout)
Lesson 18: Maximum and Minimum Values (handout)Lesson 18: Maximum and Minimum Values (handout)
Lesson 18: Maximum and Minimum Values (handout)Matthew Leingang
 

More from Matthew Leingang (20)

Making Lesson Plans
Making Lesson PlansMaking Lesson Plans
Making Lesson Plans
 
Streamlining assessment, feedback, and archival with auto-multiple-choice
Streamlining assessment, feedback, and archival with auto-multiple-choiceStreamlining assessment, feedback, and archival with auto-multiple-choice
Streamlining assessment, feedback, and archival with auto-multiple-choice
 
Electronic Grading of Paper Assessments
Electronic Grading of Paper AssessmentsElectronic Grading of Paper Assessments
Electronic Grading of Paper Assessments
 
Lesson 26: The Fundamental Theorem of Calculus (slides)
Lesson 26: The Fundamental Theorem of Calculus (slides)Lesson 26: The Fundamental Theorem of Calculus (slides)
Lesson 26: The Fundamental Theorem of Calculus (slides)
 
Lesson 27: Integration by Substitution (handout)
Lesson 27: Integration by Substitution (handout)Lesson 27: Integration by Substitution (handout)
Lesson 27: Integration by Substitution (handout)
 
Lesson 26: The Fundamental Theorem of Calculus (handout)
Lesson 26: The Fundamental Theorem of Calculus (handout)Lesson 26: The Fundamental Theorem of Calculus (handout)
Lesson 26: The Fundamental Theorem of Calculus (handout)
 
Lesson 25: Evaluating Definite Integrals (slides)
Lesson 25: Evaluating Definite Integrals (slides)Lesson 25: Evaluating Definite Integrals (slides)
Lesson 25: Evaluating Definite Integrals (slides)
 
Lesson 25: Evaluating Definite Integrals (handout)
Lesson 25: Evaluating Definite Integrals (handout)Lesson 25: Evaluating Definite Integrals (handout)
Lesson 25: Evaluating Definite Integrals (handout)
 
Lesson 24: Areas and Distances, The Definite Integral (handout)
Lesson 24: Areas and Distances, The Definite Integral (handout)Lesson 24: Areas and Distances, The Definite Integral (handout)
Lesson 24: Areas and Distances, The Definite Integral (handout)
 
Lesson 24: Areas and Distances, The Definite Integral (slides)
Lesson 24: Areas and Distances, The Definite Integral (slides)Lesson 24: Areas and Distances, The Definite Integral (slides)
Lesson 24: Areas and Distances, The Definite Integral (slides)
 
Lesson 23: Antiderivatives (slides)
Lesson 23: Antiderivatives (slides)Lesson 23: Antiderivatives (slides)
Lesson 23: Antiderivatives (slides)
 
Lesson 23: Antiderivatives (slides)
Lesson 23: Antiderivatives (slides)Lesson 23: Antiderivatives (slides)
Lesson 23: Antiderivatives (slides)
 
Lesson 22: Optimization Problems (slides)
Lesson 22: Optimization Problems (slides)Lesson 22: Optimization Problems (slides)
Lesson 22: Optimization Problems (slides)
 
Lesson 22: Optimization Problems (handout)
Lesson 22: Optimization Problems (handout)Lesson 22: Optimization Problems (handout)
Lesson 22: Optimization Problems (handout)
 
Lesson 21: Curve Sketching (handout)
Lesson 21: Curve Sketching (handout)Lesson 21: Curve Sketching (handout)
Lesson 21: Curve Sketching (handout)
 
Lesson 20: Derivatives and the Shapes of Curves (slides)
Lesson 20: Derivatives and the Shapes of Curves (slides)Lesson 20: Derivatives and the Shapes of Curves (slides)
Lesson 20: Derivatives and the Shapes of Curves (slides)
 
Lesson 20: Derivatives and the Shapes of Curves (handout)
Lesson 20: Derivatives and the Shapes of Curves (handout)Lesson 20: Derivatives and the Shapes of Curves (handout)
Lesson 20: Derivatives and the Shapes of Curves (handout)
 
Lesson 19: The Mean Value Theorem (slides)
Lesson 19: The Mean Value Theorem (slides)Lesson 19: The Mean Value Theorem (slides)
Lesson 19: The Mean Value Theorem (slides)
 
Lesson 17: Indeterminate forms and l'Hôpital's Rule (slides)
Lesson 17: Indeterminate forms and l'Hôpital's Rule (slides)Lesson 17: Indeterminate forms and l'Hôpital's Rule (slides)
Lesson 17: Indeterminate forms and l'Hôpital's Rule (slides)
 
Lesson 18: Maximum and Minimum Values (handout)
Lesson 18: Maximum and Minimum Values (handout)Lesson 18: Maximum and Minimum Values (handout)
Lesson 18: Maximum and Minimum Values (handout)
 

Recently uploaded

08448380779 Call Girls In Civil Lines Women Seeking Men
08448380779 Call Girls In Civil Lines Women Seeking Men08448380779 Call Girls In Civil Lines Women Seeking Men
08448380779 Call Girls In Civil Lines Women Seeking MenDelhi Call girls
 
Strategize a Smooth Tenant-to-tenant Migration and Copilot Takeoff
Strategize a Smooth Tenant-to-tenant Migration and Copilot TakeoffStrategize a Smooth Tenant-to-tenant Migration and Copilot Takeoff
Strategize a Smooth Tenant-to-tenant Migration and Copilot Takeoffsammart93
 
Evaluating the top large language models.pdf
Evaluating the top large language models.pdfEvaluating the top large language models.pdf
Evaluating the top large language models.pdfChristopherTHyatt
 
presentation ICT roal in 21st century education
presentation ICT roal in 21st century educationpresentation ICT roal in 21st century education
presentation ICT roal in 21st century educationjfdjdjcjdnsjd
 
04-2024-HHUG-Sales-and-Marketing-Alignment.pptx
04-2024-HHUG-Sales-and-Marketing-Alignment.pptx04-2024-HHUG-Sales-and-Marketing-Alignment.pptx
04-2024-HHUG-Sales-and-Marketing-Alignment.pptxHampshireHUG
 
Understanding Discord NSFW Servers A Guide for Responsible Users.pdf
Understanding Discord NSFW Servers A Guide for Responsible Users.pdfUnderstanding Discord NSFW Servers A Guide for Responsible Users.pdf
Understanding Discord NSFW Servers A Guide for Responsible Users.pdfUK Journal
 
Boost PC performance: How more available memory can improve productivity
Boost PC performance: How more available memory can improve productivityBoost PC performance: How more available memory can improve productivity
Boost PC performance: How more available memory can improve productivityPrincipled Technologies
 
Bajaj Allianz Life Insurance Company - Insurer Innovation Award 2024
Bajaj Allianz Life Insurance Company - Insurer Innovation Award 2024Bajaj Allianz Life Insurance Company - Insurer Innovation Award 2024
Bajaj Allianz Life Insurance Company - Insurer Innovation Award 2024The Digital Insurer
 
Mastering MySQL Database Architecture: Deep Dive into MySQL Shell and MySQL R...
Mastering MySQL Database Architecture: Deep Dive into MySQL Shell and MySQL R...Mastering MySQL Database Architecture: Deep Dive into MySQL Shell and MySQL R...
Mastering MySQL Database Architecture: Deep Dive into MySQL Shell and MySQL R...Miguel Araújo
 
Exploring the Future Potential of AI-Enabled Smartphone Processors
Exploring the Future Potential of AI-Enabled Smartphone ProcessorsExploring the Future Potential of AI-Enabled Smartphone Processors
Exploring the Future Potential of AI-Enabled Smartphone Processorsdebabhi2
 
Powerful Google developer tools for immediate impact! (2023-24 C)
Powerful Google developer tools for immediate impact! (2023-24 C)Powerful Google developer tools for immediate impact! (2023-24 C)
Powerful Google developer tools for immediate impact! (2023-24 C)wesley chun
 
Presentation on how to chat with PDF using ChatGPT code interpreter
Presentation on how to chat with PDF using ChatGPT code interpreterPresentation on how to chat with PDF using ChatGPT code interpreter
Presentation on how to chat with PDF using ChatGPT code interpreternaman860154
 
08448380779 Call Girls In Diplomatic Enclave Women Seeking Men
08448380779 Call Girls In Diplomatic Enclave Women Seeking Men08448380779 Call Girls In Diplomatic Enclave Women Seeking Men
08448380779 Call Girls In Diplomatic Enclave Women Seeking MenDelhi Call girls
 
🐬 The future of MySQL is Postgres 🐘
🐬  The future of MySQL is Postgres   🐘🐬  The future of MySQL is Postgres   🐘
🐬 The future of MySQL is Postgres 🐘RTylerCroy
 
Apidays Singapore 2024 - Building Digital Trust in a Digital Economy by Veron...
Apidays Singapore 2024 - Building Digital Trust in a Digital Economy by Veron...Apidays Singapore 2024 - Building Digital Trust in a Digital Economy by Veron...
Apidays Singapore 2024 - Building Digital Trust in a Digital Economy by Veron...apidays
 
The 7 Things I Know About Cyber Security After 25 Years | April 2024
The 7 Things I Know About Cyber Security After 25 Years | April 2024The 7 Things I Know About Cyber Security After 25 Years | April 2024
The 7 Things I Know About Cyber Security After 25 Years | April 2024Rafal Los
 
Finology Group – Insurtech Innovation Award 2024
Finology Group – Insurtech Innovation Award 2024Finology Group – Insurtech Innovation Award 2024
Finology Group – Insurtech Innovation Award 2024The Digital Insurer
 
2024: Domino Containers - The Next Step. News from the Domino Container commu...
2024: Domino Containers - The Next Step. News from the Domino Container commu...2024: Domino Containers - The Next Step. News from the Domino Container commu...
2024: Domino Containers - The Next Step. News from the Domino Container commu...Martijn de Jong
 
Strategies for Unlocking Knowledge Management in Microsoft 365 in the Copilot...
Strategies for Unlocking Knowledge Management in Microsoft 365 in the Copilot...Strategies for Unlocking Knowledge Management in Microsoft 365 in the Copilot...
Strategies for Unlocking Knowledge Management in Microsoft 365 in the Copilot...Drew Madelung
 
How to convert PDF to text with Nanonets
How to convert PDF to text with NanonetsHow to convert PDF to text with Nanonets
How to convert PDF to text with Nanonetsnaman860154
 

Recently uploaded (20)

08448380779 Call Girls In Civil Lines Women Seeking Men
08448380779 Call Girls In Civil Lines Women Seeking Men08448380779 Call Girls In Civil Lines Women Seeking Men
08448380779 Call Girls In Civil Lines Women Seeking Men
 
Strategize a Smooth Tenant-to-tenant Migration and Copilot Takeoff
Strategize a Smooth Tenant-to-tenant Migration and Copilot TakeoffStrategize a Smooth Tenant-to-tenant Migration and Copilot Takeoff
Strategize a Smooth Tenant-to-tenant Migration and Copilot Takeoff
 
Evaluating the top large language models.pdf
Evaluating the top large language models.pdfEvaluating the top large language models.pdf
Evaluating the top large language models.pdf
 
presentation ICT roal in 21st century education
presentation ICT roal in 21st century educationpresentation ICT roal in 21st century education
presentation ICT roal in 21st century education
 
04-2024-HHUG-Sales-and-Marketing-Alignment.pptx
04-2024-HHUG-Sales-and-Marketing-Alignment.pptx04-2024-HHUG-Sales-and-Marketing-Alignment.pptx
04-2024-HHUG-Sales-and-Marketing-Alignment.pptx
 
Understanding Discord NSFW Servers A Guide for Responsible Users.pdf
Understanding Discord NSFW Servers A Guide for Responsible Users.pdfUnderstanding Discord NSFW Servers A Guide for Responsible Users.pdf
Understanding Discord NSFW Servers A Guide for Responsible Users.pdf
 
Boost PC performance: How more available memory can improve productivity
Boost PC performance: How more available memory can improve productivityBoost PC performance: How more available memory can improve productivity
Boost PC performance: How more available memory can improve productivity
 
Bajaj Allianz Life Insurance Company - Insurer Innovation Award 2024
Bajaj Allianz Life Insurance Company - Insurer Innovation Award 2024Bajaj Allianz Life Insurance Company - Insurer Innovation Award 2024
Bajaj Allianz Life Insurance Company - Insurer Innovation Award 2024
 
Mastering MySQL Database Architecture: Deep Dive into MySQL Shell and MySQL R...
Mastering MySQL Database Architecture: Deep Dive into MySQL Shell and MySQL R...Mastering MySQL Database Architecture: Deep Dive into MySQL Shell and MySQL R...
Mastering MySQL Database Architecture: Deep Dive into MySQL Shell and MySQL R...
 
Exploring the Future Potential of AI-Enabled Smartphone Processors
Exploring the Future Potential of AI-Enabled Smartphone ProcessorsExploring the Future Potential of AI-Enabled Smartphone Processors
Exploring the Future Potential of AI-Enabled Smartphone Processors
 
Powerful Google developer tools for immediate impact! (2023-24 C)
Powerful Google developer tools for immediate impact! (2023-24 C)Powerful Google developer tools for immediate impact! (2023-24 C)
Powerful Google developer tools for immediate impact! (2023-24 C)
 
Presentation on how to chat with PDF using ChatGPT code interpreter
Presentation on how to chat with PDF using ChatGPT code interpreterPresentation on how to chat with PDF using ChatGPT code interpreter
Presentation on how to chat with PDF using ChatGPT code interpreter
 
08448380779 Call Girls In Diplomatic Enclave Women Seeking Men
08448380779 Call Girls In Diplomatic Enclave Women Seeking Men08448380779 Call Girls In Diplomatic Enclave Women Seeking Men
08448380779 Call Girls In Diplomatic Enclave Women Seeking Men
 
🐬 The future of MySQL is Postgres 🐘
🐬  The future of MySQL is Postgres   🐘🐬  The future of MySQL is Postgres   🐘
🐬 The future of MySQL is Postgres 🐘
 
Apidays Singapore 2024 - Building Digital Trust in a Digital Economy by Veron...
Apidays Singapore 2024 - Building Digital Trust in a Digital Economy by Veron...Apidays Singapore 2024 - Building Digital Trust in a Digital Economy by Veron...
Apidays Singapore 2024 - Building Digital Trust in a Digital Economy by Veron...
 
The 7 Things I Know About Cyber Security After 25 Years | April 2024
The 7 Things I Know About Cyber Security After 25 Years | April 2024The 7 Things I Know About Cyber Security After 25 Years | April 2024
The 7 Things I Know About Cyber Security After 25 Years | April 2024
 
Finology Group – Insurtech Innovation Award 2024
Finology Group – Insurtech Innovation Award 2024Finology Group – Insurtech Innovation Award 2024
Finology Group – Insurtech Innovation Award 2024
 
2024: Domino Containers - The Next Step. News from the Domino Container commu...
2024: Domino Containers - The Next Step. News from the Domino Container commu...2024: Domino Containers - The Next Step. News from the Domino Container commu...
2024: Domino Containers - The Next Step. News from the Domino Container commu...
 
Strategies for Unlocking Knowledge Management in Microsoft 365 in the Copilot...
Strategies for Unlocking Knowledge Management in Microsoft 365 in the Copilot...Strategies for Unlocking Knowledge Management in Microsoft 365 in the Copilot...
Strategies for Unlocking Knowledge Management in Microsoft 365 in the Copilot...
 
How to convert PDF to text with Nanonets
How to convert PDF to text with NanonetsHow to convert PDF to text with Nanonets
How to convert PDF to text with Nanonets
 

Lesson 27: Integration by Substitution (Section 041 slides)

  • 1. Section 5.5 Integration by Substitution V63.0121.041, Calculus I New York University December 13, 2010 Announcements ”Wednesday”, December 15: Review, Movie Monday, December 20, 12:00pm–1:50pm: Final Exam
  • 2. Announcements ”Wednesday”, December 15: Review, Movie Monday, December 20, 12:00pm–1:50pm: Final Exam V63.0121.041, Calculus I (NYU) Section 5.5 Integration by Substitution December 13, 2010 2 / 37
  • 3. Resurrection Policy If your final score beats your midterm score, we will add 10% to its weight, and subtract 10% from the midterm weight. Image credit: Scott Beale / Laughing Squid V63.0121.041, Calculus I (NYU) Section 5.5 Integration by Substitution December 13, 2010 3 / 37
  • 4. Objectives Given an integral and a substitution, transform the integral into an equivalent one using a substitution Evaluate indefinite integrals using the method of substitution. Evaluate definite integrals using the method of substitution. V63.0121.041, Calculus I (NYU) Section 5.5 Integration by Substitution December 13, 2010 4 / 37
  • 5. Outline Last Time: The Fundamental Theorem(s) of Calculus Substitution for Indefinite Integrals Theory Examples Substitution for Definite Integrals Theory Examples V63.0121.041, Calculus I (NYU) Section 5.5 Integration by Substitution December 13, 2010 5 / 37
  • 6. Differentiation and Integration as reverse processes Theorem (The Fundamental Theorem of Calculus) 1. Let f be continuous on [a, b]. Then x d f (t) dt = f (x) dx a 2. Let f be continuous on [a, b] and f = F for some other function F . Then b f (x) dx = F (b) − F (a). a V63.0121.041, Calculus I (NYU) Section 5.5 Integration by Substitution December 13, 2010 6 / 37
  • 7. Techniques of antidifferentiation? So far we know only a few rules for antidifferentiation. Some are general, like [f (x) + g (x)] dx = f (x) dx + g (x) dx V63.0121.041, Calculus I (NYU) Section 5.5 Integration by Substitution December 13, 2010 7 / 37
  • 8. Techniques of antidifferentiation? So far we know only a few rules for antidifferentiation. Some are general, like [f (x) + g (x)] dx = f (x) dx + g (x) dx Some are pretty particular, like 1 √ dx = arcsec x + C . x x2 − 1 V63.0121.041, Calculus I (NYU) Section 5.5 Integration by Substitution December 13, 2010 7 / 37
  • 9. Techniques of antidifferentiation? So far we know only a few rules for antidifferentiation. Some are general, like [f (x) + g (x)] dx = f (x) dx + g (x) dx Some are pretty particular, like 1 √ dx = arcsec x + C . x x2 − 1 What are we supposed to do with that? V63.0121.041, Calculus I (NYU) Section 5.5 Integration by Substitution December 13, 2010 7 / 37
  • 10. No straightforward system of antidifferentiation So far we don’t have any way to find 2x √ dx x2 + 1 or tan x dx. V63.0121.041, Calculus I (NYU) Section 5.5 Integration by Substitution December 13, 2010 8 / 37
  • 11. No straightforward system of antidifferentiation So far we don’t have any way to find 2x √ dx x2 + 1 or tan x dx. Luckily, we can be smart and use the “anti” version of one of the most important rules of differentiation: the chain rule. V63.0121.041, Calculus I (NYU) Section 5.5 Integration by Substitution December 13, 2010 8 / 37
  • 12. Outline Last Time: The Fundamental Theorem(s) of Calculus Substitution for Indefinite Integrals Theory Examples Substitution for Definite Integrals Theory Examples V63.0121.041, Calculus I (NYU) Section 5.5 Integration by Substitution December 13, 2010 9 / 37
  • 13. Substitution for Indefinite Integrals Example Find x √ dx. x2 +1 V63.0121.041, Calculus I (NYU) Section 5.5 Integration by Substitution December 13, 2010 10 / 37
  • 14. Substitution for Indefinite Integrals Example Find x √ dx. x2 +1 Solution Stare at this long enough and you notice the the integrand is the derivative of the expression 1 + x 2 . V63.0121.041, Calculus I (NYU) Section 5.5 Integration by Substitution December 13, 2010 10 / 37
  • 15. Say what? Solution (More slowly, now) Let g (x) = x 2 + 1. V63.0121.041, Calculus I (NYU) Section 5.5 Integration by Substitution December 13, 2010 11 / 37
  • 16. Say what? Solution (More slowly, now) Let g (x) = x 2 + 1. Then g (x) = 2x and so d 1 x g (x) = g (x) = √ dx 2 g (x) x2 +1 V63.0121.041, Calculus I (NYU) Section 5.5 Integration by Substitution December 13, 2010 11 / 37
  • 17. Say what? Solution (More slowly, now) Let g (x) = x 2 + 1. Then g (x) = 2x and so d 1 x g (x) = g (x) = √ dx 2 g (x) x2 +1 Thus x d √ dx = g (x) dx x2 + 1 dx = g (x) + C = 1 + x2 + C . V63.0121.041, Calculus I (NYU) Section 5.5 Integration by Substitution December 13, 2010 11 / 37
  • 18. Leibnizian notation FTW Solution (Same technique, new notation) Let u = x 2 + 1. V63.0121.041, Calculus I (NYU) Section 5.5 Integration by Substitution December 13, 2010 12 / 37
  • 19. Leibnizian notation FTW Solution (Same technique, new notation) √ Let u = x 2 + 1. Then du = 2x dx and 1 + x2 = u. V63.0121.041, Calculus I (NYU) Section 5.5 Integration by Substitution December 13, 2010 12 / 37
  • 20. Leibnizian notation FTW Solution (Same technique, new notation) √ Let u = x 2 + 1. Then du = 2x dx and 1 + x2 = u. So the integrand becomes completely transformed into 1 √ x dx 2 du √ 1 √ du = = x2 + 1 u 2 u V63.0121.041, Calculus I (NYU) Section 5.5 Integration by Substitution December 13, 2010 12 / 37
  • 21. Leibnizian notation FTW Solution (Same technique, new notation) √ Let u = x 2 + 1. Then du = 2x dx and 1 + x2 = u. So the integrand becomes completely transformed into 1 √ x dx 2 du √ 1 √ du = = x2 + 1 u 2 u 1 −1/2 = 2u du V63.0121.041, Calculus I (NYU) Section 5.5 Integration by Substitution December 13, 2010 12 / 37
  • 22. Leibnizian notation FTW Solution (Same technique, new notation) √ Let u = x 2 + 1. Then du = 2x dx and 1 + x2 = u. So the integrand becomes completely transformed into 1 √ x dx 2 du √ 1 √ du = = x2 + 1 u 2 u 1 −1/2 = 2u du √ = u+C = 1 + x2 + C . V63.0121.041, Calculus I (NYU) Section 5.5 Integration by Substitution December 13, 2010 12 / 37
  • 23. Useful but unsavory variation Solution (Same technique, new notation, more idiot-proof) √ Let u = x 2 + 1. Then du = 2x dx and 1 + x 2 = u. “Solve for dx:” du dx = 2x V63.0121.041, Calculus I (NYU) Section 5.5 Integration by Substitution December 13, 2010 13 / 37
  • 24. Useful but unsavory variation Solution (Same technique, new notation, more idiot-proof) √ Let u = x 2 + 1. Then du = 2x dx and 1 + x 2 = u. “Solve for dx:” du dx = 2x So the integrand becomes completely transformed into x x du √ dx = √ · x2 +1 u 2x V63.0121.041, Calculus I (NYU) Section 5.5 Integration by Substitution December 13, 2010 13 / 37
  • 25. Useful but unsavory variation Solution (Same technique, new notation, more idiot-proof) √ Let u = x 2 + 1. Then du = 2x dx and 1 + x 2 = u. “Solve for dx:” du dx = 2x So the integrand becomes completely transformed into x x du 1 √ dx = √ · = √ du x2 +1 u 2x 2 u V63.0121.041, Calculus I (NYU) Section 5.5 Integration by Substitution December 13, 2010 13 / 37
  • 26. Useful but unsavory variation Solution (Same technique, new notation, more idiot-proof) √ Let u = x 2 + 1. Then du = 2x dx and 1 + x 2 = u. “Solve for dx:” du dx = 2x So the integrand becomes completely transformed into x x du 1 √ dx = √ · = √ du x2 +1 u 2x 2 u 1 −1/2 = 2u du V63.0121.041, Calculus I (NYU) Section 5.5 Integration by Substitution December 13, 2010 13 / 37
  • 27. Useful but unsavory variation Solution (Same technique, new notation, more idiot-proof) √ Let u = x 2 + 1. Then du = 2x dx and 1 + x 2 = u. “Solve for dx:” du dx = 2x So the integrand becomes completely transformed into x x du 1 √ dx = √ · = √ du x2 +1 u 2x 2 u 1 −1/2 = 2u du √ = u+C = 1 + x2 + C . V63.0121.041, Calculus I (NYU) Section 5.5 Integration by Substitution December 13, 2010 13 / 37
  • 28. Useful but unsavory variation Solution (Same technique, new notation, more idiot-proof) √ Let u = x 2 + 1. Then du = 2x dx and 1 + x 2 = u. “Solve for dx:” du dx = 2x So the integrand becomes completely transformed into x x du 1 √ dx = √ · = √ du x2 +1 u 2x 2 u 1 −1/2 = 2u du √ = u+C = 1 + x2 + C . Mathematicians have serious issues with mixing the x and u like this. However, I can’t deny that it works. V63.0121.041, Calculus I (NYU) Section 5.5 Integration by Substitution December 13, 2010 13 / 37
  • 29. Theorem of the Day Theorem (The Substitution Rule) If u = g (x) is a differentiable function whose range is an interval I and f is continuous on I , then f (g (x))g (x) dx = f (u) du That is, if F is an antiderivative for f , then f (g (x))g (x) dx = F (g (x)) In Leibniz notation: du f (u) dx = f (u) du dx V63.0121.041, Calculus I (NYU) Section 5.5 Integration by Substitution December 13, 2010 14 / 37
  • 30. A polynomial example Example Use the substitution u = x 2 + 3 to find (x 2 + 3)3 4x dx. V63.0121.041, Calculus I (NYU) Section 5.5 Integration by Substitution December 13, 2010 15 / 37
  • 31. A polynomial example Example Use the substitution u = x 2 + 3 to find (x 2 + 3)3 4x dx. Solution If u = x 2 + 3, then du = 2x dx, and 4x dx = 2 du. So (x 2 + 3)3 4x dx V63.0121.041, Calculus I (NYU) Section 5.5 Integration by Substitution December 13, 2010 15 / 37
  • 32. A polynomial example Example Use the substitution u = x 2 + 3 to find (x 2 + 3)3 4x dx. Solution If u = x 2 + 3, then du = 2x dx, and 4x dx = 2 du. So (x 2 + 3)3 4x dx = u 3 2du = 2 u 3 du V63.0121.041, Calculus I (NYU) Section 5.5 Integration by Substitution December 13, 2010 15 / 37
  • 33. A polynomial example Example Use the substitution u = x 2 + 3 to find (x 2 + 3)3 4x dx. Solution If u = x 2 + 3, then du = 2x dx, and 4x dx = 2 du. So (x 2 + 3)3 4x dx = u 3 2du = 2 u 3 du 1 = u4 2 V63.0121.041, Calculus I (NYU) Section 5.5 Integration by Substitution December 13, 2010 15 / 37
  • 34. A polynomial example Example Use the substitution u = x 2 + 3 to find (x 2 + 3)3 4x dx. Solution If u = x 2 + 3, then du = 2x dx, and 4x dx = 2 du. So (x 2 + 3)3 4x dx = u 3 2du = 2 u 3 du 1 1 = u 4 = (x 2 + 3)4 2 2 V63.0121.041, Calculus I (NYU) Section 5.5 Integration by Substitution December 13, 2010 15 / 37
  • 35. A polynomial example, by brute force Compare this to multiplying it out: (x 2 + 3)3 4x dx V63.0121.041, Calculus I (NYU) Section 5.5 Integration by Substitution December 13, 2010 16 / 37
  • 36. A polynomial example, by brute force Compare this to multiplying it out: (x 2 + 3)3 4x dx = x 6 + 9x 4 + 27x 2 + 27 4x dx V63.0121.041, Calculus I (NYU) Section 5.5 Integration by Substitution December 13, 2010 16 / 37
  • 37. A polynomial example, by brute force Compare this to multiplying it out: (x 2 + 3)3 4x dx = x 6 + 9x 4 + 27x 2 + 27 4x dx = 4x 7 + 36x 5 + 108x 3 + 108x dx V63.0121.041, Calculus I (NYU) Section 5.5 Integration by Substitution December 13, 2010 16 / 37
  • 38. A polynomial example, by brute force Compare this to multiplying it out: (x 2 + 3)3 4x dx = x 6 + 9x 4 + 27x 2 + 27 4x dx = 4x 7 + 36x 5 + 108x 3 + 108x dx 1 = x 8 + 6x 6 + 27x 4 + 54x 2 2 V63.0121.041, Calculus I (NYU) Section 5.5 Integration by Substitution December 13, 2010 16 / 37
  • 39. A polynomial example, by brute force Compare this to multiplying it out: (x 2 + 3)3 4x dx = x 6 + 9x 4 + 27x 2 + 27 4x dx = 4x 7 + 36x 5 + 108x 3 + 108x dx 1 = x 8 + 6x 6 + 27x 4 + 54x 2 2 Which would you rather do? V63.0121.041, Calculus I (NYU) Section 5.5 Integration by Substitution December 13, 2010 16 / 37
  • 40. A polynomial example, by brute force Compare this to multiplying it out: (x 2 + 3)3 4x dx = x 6 + 9x 4 + 27x 2 + 27 4x dx = 4x 7 + 36x 5 + 108x 3 + 108x dx 1 = x 8 + 6x 6 + 27x 4 + 54x 2 2 Which would you rather do? It’s a wash for low powers V63.0121.041, Calculus I (NYU) Section 5.5 Integration by Substitution December 13, 2010 16 / 37
  • 41. A polynomial example, by brute force Compare this to multiplying it out: (x 2 + 3)3 4x dx = x 6 + 9x 4 + 27x 2 + 27 4x dx = 4x 7 + 36x 5 + 108x 3 + 108x dx 1 = x 8 + 6x 6 + 27x 4 + 54x 2 2 Which would you rather do? It’s a wash for low powers But for higher powers, it’s much easier to do substitution. V63.0121.041, Calculus I (NYU) Section 5.5 Integration by Substitution December 13, 2010 16 / 37
  • 42. Compare We have the substitution method, which, when multiplied out, gives 1 (x 2 + 3)3 4x dx = (x 2 + 3)4 2 1 8 = x + 12x 6 + 54x 4 + 108x 2 + 81 2 1 81 = x 8 + 6x 6 + 27x 4 + 54x 2 + 2 2 and the brute force method 1 (x 2 + 3)3 4x dx = x 8 + 6x 6 + 27x 4 + 54x 2 2 Is there a difference? Is this a problem? V63.0121.041, Calculus I (NYU) Section 5.5 Integration by Substitution December 13, 2010 17 / 37
  • 43. Compare We have the substitution method, which, when multiplied out, gives 1 (x 2 + 3)3 4x dx = (x 2 + 3)4 + C 2 1 8 = x + 12x 6 + 54x 4 + 108x 2 + 81 + C 2 1 81 = x 8 + 6x 6 + 27x 4 + 54x 2 + +C 2 2 and the brute force method 1 (x 2 + 3)3 4x dx = x 8 + 6x 6 + 27x 4 + 54x 2 + C 2 Is there a difference? Is this a problem? No, that’s what +C means! V63.0121.041, Calculus I (NYU) Section 5.5 Integration by Substitution December 13, 2010 17 / 37
  • 44. A slick example Example Find tan x dx. V63.0121.041, Calculus I (NYU) Section 5.5 Integration by Substitution December 13, 2010 18 / 37
  • 45. A slick example Example sin x Find tan x dx. (Hint: tan x = ) cos x V63.0121.041, Calculus I (NYU) Section 5.5 Integration by Substitution December 13, 2010 18 / 37
  • 46. A slick example Example sin x Find tan x dx. (Hint: tan x = ) cos x Solution Let u = cos x . Then du = − sin x dx . So sin x tan x dx = dx cos x V63.0121.041, Calculus I (NYU) Section 5.5 Integration by Substitution December 13, 2010 18 / 37
  • 47. A slick example Example sin x Find tan x dx. (Hint: tan x = ) cos x Solution Let u = cos x . Then du = − sin x dx . So sin x tan x dx = dx cos x V63.0121.041, Calculus I (NYU) Section 5.5 Integration by Substitution December 13, 2010 18 / 37
  • 48. A slick example Example sin x Find tan x dx. (Hint: tan x = ) cos x Solution Let u = cos x . Then du = − sin x dx . So sin x tan x dx = dx cos x V63.0121.041, Calculus I (NYU) Section 5.5 Integration by Substitution December 13, 2010 18 / 37
  • 49. A slick example Example sin x Find tan x dx. (Hint: tan x = ) cos x Solution Let u = cos x . Then du = − sin x dx . So sin x 1 tan x dx = dx = − du cos x u V63.0121.041, Calculus I (NYU) Section 5.5 Integration by Substitution December 13, 2010 18 / 37
  • 50. A slick example Example sin x Find tan x dx. (Hint: tan x = ) cos x Solution Let u = cos x . Then du = − sin x dx . So sin x 1 tan x dx = dx = − du cos x u = − ln |u| + C V63.0121.041, Calculus I (NYU) Section 5.5 Integration by Substitution December 13, 2010 18 / 37
  • 51. A slick example Example sin x Find tan x dx. (Hint: tan x = ) cos x Solution Let u = cos x . Then du = − sin x dx . So sin x 1 tan x dx = dx = − du cos x u = − ln |u| + C = − ln | cos x| + C = ln | sec x| + C V63.0121.041, Calculus I (NYU) Section 5.5 Integration by Substitution December 13, 2010 18 / 37
  • 52. Can you do it another way? Example sin x Find tan x dx. (Hint: tan x = ) cos x V63.0121.041, Calculus I (NYU) Section 5.5 Integration by Substitution December 13, 2010 19 / 37
  • 53. Can you do it another way? Example sin x Find tan x dx. (Hint: tan x = ) cos x Solution du Let u = sin x. Then du = cos x dx and so dx = . cos x V63.0121.041, Calculus I (NYU) Section 5.5 Integration by Substitution December 13, 2010 19 / 37
  • 54. Can you do it another way? Example sin x Find tan x dx. (Hint: tan x = ) cos x Solution du Let u = sin x. Then du = cos x dx and so dx = . cos x sin x u du tan x dx = dx = cos x cos x cos x u du u du u du = = = cos 2x 1 − sin2 x 1 − u2 At this point, although it’s possible to proceed, we should probably back up and see if the other way works quicker (it does). V63.0121.041, Calculus I (NYU) Section 5.5 Integration by Substitution December 13, 2010 19 / 37
  • 55. For those who really must know all Solution (Continued, with algebra help) Let y = 1 − u 2 , so dy = −2u du. Then u du u dy tan x dx = 2 = 1−u y −2u 1 dy 1 1 =− = − ln |y | + C = − ln 1 − u 2 + C 2 y 2 2 1 1 = ln √ + C = ln +C 1 − u2 1 − sin2 x 1 = ln + C = ln |sec x| + C |cos x| There are other ways to do it, too. V63.0121.041, Calculus I (NYU) Section 5.5 Integration by Substitution December 13, 2010 20 / 37
  • 56. Outline Last Time: The Fundamental Theorem(s) of Calculus Substitution for Indefinite Integrals Theory Examples Substitution for Definite Integrals Theory Examples V63.0121.041, Calculus I (NYU) Section 5.5 Integration by Substitution December 13, 2010 21 / 37
  • 57. Substitution for Definite Integrals Theorem (The Substitution Rule for Definite Integrals) If g is continuous and f is continuous on the range of u = g (x), then b g (b) f (g (x))g (x) dx = f (u) du. a g (a) V63.0121.041, Calculus I (NYU) Section 5.5 Integration by Substitution December 13, 2010 22 / 37
  • 58. Substitution for Definite Integrals Theorem (The Substitution Rule for Definite Integrals) If g is continuous and f is continuous on the range of u = g (x), then b g (b) f (g (x))g (x) dx = f (u) du. a g (a) Why the change in the limits? The integral on the left happens in “x-land” The integral on the right happens in “u-land”, so the limits need to be u-values To get from x to u, apply g V63.0121.041, Calculus I (NYU) Section 5.5 Integration by Substitution December 13, 2010 22 / 37
  • 59. Example π Compute cos2 x sin x dx. 0 V63.0121.041, Calculus I (NYU) Section 5.5 Integration by Substitution December 13, 2010 23 / 37
  • 60. Example π Compute cos2 x sin x dx. 0 Solution (Slow Way) First compute the indefinite integral cos2 x sin x dx and then evaluate. Let u = cos x . Then du = − sin x dx and cos2 x sin x dx V63.0121.041, Calculus I (NYU) Section 5.5 Integration by Substitution December 13, 2010 23 / 37
  • 61. Example π Compute cos2 x sin x dx. 0 Solution (Slow Way) First compute the indefinite integral cos2 x sin x dx and then evaluate. Let u = cos x . Then du = − sin x dx and cos2 x sin x dx V63.0121.041, Calculus I (NYU) Section 5.5 Integration by Substitution December 13, 2010 23 / 37
  • 62. Example π Compute cos2 x sin x dx. 0 Solution (Slow Way) First compute the indefinite integral cos2 x sin x dx and then evaluate. Let u = cos x . Then du = − sin x dx and cos2 x sin x dx V63.0121.041, Calculus I (NYU) Section 5.5 Integration by Substitution December 13, 2010 23 / 37
  • 63. Example π Compute cos2 x sin x dx. 0 Solution (Slow Way) First compute the indefinite integral cos2 x sin x dx and then evaluate. Let u = cos x . Then du = − sin x dx and cos2 x sin x dx = − u 2 du V63.0121.041, Calculus I (NYU) Section 5.5 Integration by Substitution December 13, 2010 23 / 37
  • 64. Example π Compute cos2 x sin x dx. 0 Solution (Slow Way) First compute the indefinite integral cos2 x sin x dx and then evaluate. Let u = cos x . Then du = − sin x dx and cos2 x sin x dx = − u 2 du = − 3 u 3 + C = − 1 cos3 x + C . 1 3 V63.0121.041, Calculus I (NYU) Section 5.5 Integration by Substitution December 13, 2010 23 / 37
  • 65. Example π Compute cos2 x sin x dx. 0 Solution (Slow Way) First compute the indefinite integral cos2 x sin x dx and then evaluate. Let u = cos x . Then du = − sin x dx and cos2 x sin x dx = − u 2 du = − 3 u 3 + C = − 1 cos3 x + C . 1 3 Therefore π π 1 cos2 x sin x dx = − cos3 x 0 3 0 V63.0121.041, Calculus I (NYU) Section 5.5 Integration by Substitution December 13, 2010 23 / 37
  • 66. Example π Compute cos2 x sin x dx. 0 Solution (Slow Way) First compute the indefinite integral cos2 x sin x dx and then evaluate. Let u = cos x . Then du = − sin x dx and cos2 x sin x dx = − u 2 du = − 3 u 3 + C = − 1 cos3 x + C . 1 3 Therefore π π 1 1 cos2 x sin x dx = − cos3 x =− (−1)3 − 13 0 3 0 3 V63.0121.041, Calculus I (NYU) Section 5.5 Integration by Substitution December 13, 2010 23 / 37
  • 67. Example π Compute cos2 x sin x dx. 0 Solution (Slow Way) First compute the indefinite integral cos2 x sin x dx and then evaluate. Let u = cos x . Then du = − sin x dx and cos2 x sin x dx = − u 2 du = − 3 u 3 + C = − 1 cos3 x + C . 1 3 Therefore π π 1 1 2 cos2 x sin x dx = − cos3 x =− (−1)3 − 13 = . 0 3 0 3 3 V63.0121.041, Calculus I (NYU) Section 5.5 Integration by Substitution December 13, 2010 23 / 37
  • 68. Definite-ly Quicker Solution (Fast Way) Do both the substitution and the evaluation at the same time. Let u = cos x. Then du = − sin x dx, u(0) = 1 and u(π) = −1 . So π cos2 x sin x dx 0 V63.0121.041, Calculus I (NYU) Section 5.5 Integration by Substitution December 13, 2010 24 / 37
  • 69. Definite-ly Quicker Solution (Fast Way) Do both the substitution and the evaluation at the same time. Let u = cos x. Then du = − sin x dx, u(0) = 1 and u(π) = −1 . So π cos2 x sin x dx 0 V63.0121.041, Calculus I (NYU) Section 5.5 Integration by Substitution December 13, 2010 24 / 37
  • 70. Definite-ly Quicker Solution (Fast Way) Do both the substitution and the evaluation at the same time. Let u = cos x. Then du = − sin x dx, u(0) = 1 and u(π) = −1 . So π cos2 x sin x dx 0 V63.0121.041, Calculus I (NYU) Section 5.5 Integration by Substitution December 13, 2010 24 / 37
  • 71. Definite-ly Quicker Solution (Fast Way) Do both the substitution and the evaluation at the same time. Let u = cos x. Then du = − sin x dx, u(0) = 1 and u(π) = −1 . So π −1 1 cos2 x sin x dx = −u 2 du = u 2 du 0 1 −1 V63.0121.041, Calculus I (NYU) Section 5.5 Integration by Substitution December 13, 2010 24 / 37
  • 72. Definite-ly Quicker Solution (Fast Way) Do both the substitution and the evaluation at the same time. Let u = cos x. Then du = − sin x dx, u(0) = 1 and u(π) = −1 . So π −1 1 cos2 x sin x dx = −u 2 du = u 2 du 0 1 −1 1 1 3 1 2 = u = 1 − (−1) = 3 −1 3 3 V63.0121.041, Calculus I (NYU) Section 5.5 Integration by Substitution December 13, 2010 24 / 37
  • 73. Definite-ly Quicker Solution (Fast Way) Do both the substitution and the evaluation at the same time. Let u = cos x. Then du = − sin x dx, u(0) = 1 and u(π) = −1 . So π −1 1 cos2 x sin x dx = −u 2 du = u 2 du 0 1 −1 1 1 3 1 2 = u = 1 − (−1) = 3 −1 3 3 The advantage to the “fast way” is that you completely transform the integral into something simpler and don’t have to go back to the original variable (x). V63.0121.041, Calculus I (NYU) Section 5.5 Integration by Substitution December 13, 2010 24 / 37
  • 74. Definite-ly Quicker Solution (Fast Way) Do both the substitution and the evaluation at the same time. Let u = cos x. Then du = − sin x dx, u(0) = 1 and u(π) = −1 . So π −1 1 cos2 x sin x dx = −u 2 du = u 2 du 0 1 −1 1 1 3 1 2 = u = 1 − (−1) = 3 −1 3 3 The advantage to the “fast way” is that you completely transform the integral into something simpler and don’t have to go back to the original variable (x). But the slow way is just as reliable. V63.0121.041, Calculus I (NYU) Section 5.5 Integration by Substitution December 13, 2010 24 / 37
  • 75. An exponential example Example √ ln 8 Find √ e 2x e 2x + 1 dx ln 3 V63.0121.041, Calculus I (NYU) Section 5.5 Integration by Substitution December 13, 2010 25 / 37
  • 76. An exponential example Example √ ln 8 Find √ e 2x e 2x + 1 dx ln 3 Solution Let u = e 2x , so du = 2e 2x dx. We have √ ln 8 8√ 1 √ e 2x e 2x + 1 dx = u + 1 du ln 3 2 3 V63.0121.041, Calculus I (NYU) Section 5.5 Integration by Substitution December 13, 2010 25 / 37
  • 77. About those limits Since √ √ 2 e 2(ln 3) = e ln 3 = e ln 3 = 3 we have √ ln 8 8√ 1 √ e 2x e 2x + 1 dx = u + 1 du ln 3 2 3 V63.0121.041, Calculus I (NYU) Section 5.5 Integration by Substitution December 13, 2010 26 / 37
  • 78. An exponential example Example √ ln 8 Find √ e 2x e 2x + 1 dx ln 3 Solution Let u = e 2x , so du = 2e 2x dx. We have √ ln 8 8√ 1 √ e 2x e 2x + 1 dx = u + 1 du ln 3 2 3 Now let y = u + 1, dy = du. So 8√ 9 9 1 1 √ 1 u + 1 du = y dy = y 1/2 dy 2 3 2 4 2 4 9 1 2 1 19 = · y 3/2 = (27 − 8) = 2 3 4 3 3 V63.0121.041, Calculus I (NYU) Section 5.5 Integration by Substitution December 13, 2010 27 / 37
  • 79. About those fractional powers We have 93/2 = (91/2 )3 = 33 = 27 43/2 = (41/2 )3 = 23 = 8 so 9 9 1 1 2 3/2 1 19 y 1/2 dy = · y = (27 − 8) = 2 4 2 3 4 3 3 V63.0121.041, Calculus I (NYU) Section 5.5 Integration by Substitution December 13, 2010 28 / 37
  • 80. An exponential example Example √ ln 8 Find √ e 2x e 2x + 1 dx ln 3 Solution Let u = e 2x , so du = 2e 2x dx. We have √ ln 8 8√ 1 √ e 2x e 2x + 1 dx = u + 1 du ln 3 2 3 Now let y = u + 1, dy = du. So 8√ 9 9 1 1 √ 1 u + 1 du = y dy = y 1/2 dy 2 3 2 4 2 4 9 1 2 1 19 = · y 3/2 = (27 − 8) = 2 3 4 3 3 V63.0121.041, Calculus I (NYU) Section 5.5 Integration by Substitution December 13, 2010 29 / 37
  • 81. Another way to skin that cat Example √ ln 8 Find √ e 2x e 2x + 1 dx ln 3 Solution Let u = e 2x + 1, V63.0121.041, Calculus I (NYU) Section 5.5 Integration by Substitution December 13, 2010 30 / 37
  • 82. Another way to skin that cat Example √ ln 8 Find √ e 2x e 2x + 1 dx ln 3 Solution Let u = e 2x + 1,so that du = 2e 2x dx. V63.0121.041, Calculus I (NYU) Section 5.5 Integration by Substitution December 13, 2010 30 / 37
  • 83. Another way to skin that cat Example √ ln 8 Find √ e 2x e 2x + 1 dx ln 3 Solution Let u = e 2x + 1,so that du = 2e 2x dx. Then √ ln 8 9√ 1 √ e 2x e 2x + 1 dx = u du ln 3 2 4 V63.0121.041, Calculus I (NYU) Section 5.5 Integration by Substitution December 13, 2010 30 / 37
  • 84. Another way to skin that cat Example √ ln 8 Find √ e 2x e 2x + 1 dx ln 3 Solution Let u = e 2x + 1,so that du = 2e 2x dx. Then √ ln 8 9√ 1 √ e 2x e 2x + 1 dx = u du ln 3 2 4 9 1 = u 3/2 3 4 V63.0121.041, Calculus I (NYU) Section 5.5 Integration by Substitution December 13, 2010 30 / 37
  • 85. Another way to skin that cat Example √ ln 8 Find √ e 2x e 2x + 1 dx ln 3 Solution Let u = e 2x + 1,so that du = 2e 2x dx. Then √ ln 8 9√ 1 √ e 2x e 2x + 1 dx = u du ln 3 2 4 1 3/2 9 = u 3 4 1 19 = (27 − 8) = 3 3 V63.0121.041, Calculus I (NYU) Section 5.5 Integration by Substitution December 13, 2010 30 / 37
  • 86. A third skinned cat Example √ ln 8 Find √ e 2x e 2x + 1 dx ln 3 Solution Let u = e 2x + 1, so that u 2 = e 2x + 1 V63.0121.041, Calculus I (NYU) Section 5.5 Integration by Substitution December 13, 2010 31 / 37
  • 87. A third skinned cat Example √ ln 8 Find √ e 2x e 2x + 1 dx ln 3 Solution Let u = e 2x + 1, so that u 2 = e 2x + 1 =⇒ 2u du = 2e 2x dx V63.0121.041, Calculus I (NYU) Section 5.5 Integration by Substitution December 13, 2010 31 / 37
  • 88. A third skinned cat Example √ ln 8 Find √ e 2x e 2x + 1 dx ln 3 Solution Let u = e 2x + 1, so that u 2 = e 2x + 1 =⇒ 2u du = 2e 2x dx Thus √ ln 8 3 3 1 3 19 √ = u · u du = u = ln 3 2 3 2 3 V63.0121.041, Calculus I (NYU) Section 5.5 Integration by Substitution December 13, 2010 31 / 37
  • 89. A Trigonometric Example Example Find 3π/2 θ θ cot5 sec2 dθ. π 6 6 V63.0121.041, Calculus I (NYU) Section 5.5 Integration by Substitution December 13, 2010 32 / 37
  • 90. A Trigonometric Example Example Find 3π/2 θ θ cot5 sec2 dθ. π 6 6 Before we dive in, think about: What “easy” substitutions might help? Which of the trig functions suggests a substitution? V63.0121.041, Calculus I (NYU) Section 5.5 Integration by Substitution December 13, 2010 32 / 37
  • 91. Solution θ 1 Let ϕ = . Then dϕ = dθ. 6 6 3π/2 π/4 θ θ cot5 sec2 dθ = 6 cot5 ϕ sec2 ϕ dϕ π 6 6 π/6 π/4 sec2 ϕ dϕ =6 π/6 tan5 ϕ V63.0121.041, Calculus I (NYU) Section 5.5 Integration by Substitution December 13, 2010 33 / 37
  • 92. Solution θ 1 Let ϕ = . Then dϕ = dθ. 6 6 3π/2 π/4 θ θ cot5 sec2 dθ = 6 cot5 ϕ sec2 ϕ dϕ π 6 6 π/6 π/4 sec2 ϕ dϕ =6 π/6 tan5 ϕ Now let u = tan ϕ. So du = sec2 ϕ dϕ, and π/4 1 sec2 ϕ dϕ −5 6 =6 √ u du π/6 tan5 ϕ 1/ 3 1 1 3 =6 − u −4 √ = [9 − 1] = 12. 4 1/ 3 2 V63.0121.041, Calculus I (NYU) Section 5.5 Integration by Substitution December 13, 2010 33 / 37
  • 93. Graphs 3π/2 π/4 θ θ cot 5 sec 2 dθ 6 cot5 ϕ sec2 ϕ dϕ π 6 6 π/6 y y θ ϕ 3π π ππ 2 64 The areas of these two regions are the same. V63.0121.041, Calculus I (NYU) Section 5.5 Integration by Substitution December 13, 2010 35 / 37
  • 94. Graphs π/4 1 −5 6 cot5 ϕ sec2 ϕ dϕ √ 6u du π/6 1/ 3 y y ϕ u ππ 1 1 64 √ 3 The areas of these two regions are the same. V63.0121.041, Calculus I (NYU) Section 5.5 Integration by Substitution December 13, 2010 36 / 37
  • 95. Summary If F is an antiderivative for f , then: f (g (x))g (x) dx = F (g (x)) If F is an antiderivative for f , which is continuous on the range of g , then: b g (b) f (g (x))g (x) dx = f (u) du = F (g (b)) − F (g (a)) a g (a) Antidifferentiation in general and substitution in particular is a “nonlinear” problem that needs practice, intuition, and perserverance The whole antidifferentiation story is in Chapter 6 V63.0121.041, Calculus I (NYU) Section 5.5 Integration by Substitution December 13, 2010 37 / 37